Você está na página 1de 444
NN Sane -CO-ORDINATE rl 0) 144 ‘CONTENTS Q Chap. The Point 2._The Straight Line 3._Two or More Straight Lines. 4.__Change of Axes S._The Circle 6. Two or More Circles ‘Lhe Parabola 8._The Ellipse 9._Hyperbola 10, The General Equation of the Second Degree and Tracing of Conics ERERERGEB-E GET 100 OUT OF 100 MARKS ‘The book has been written to guide the students in the examination. ITENSURES 100% MARKS The student is advised to take this book only as a guide. He should solve himself as many questions as possible, UHSpecial Features ; 1. Solutions to all types of important questions that may be asked in the examination. 2. Questions from up-to-date papers of various Universities are inserted at proper places. 3. The book is useful to bright as well as weak students alike. 4, Approach to the subject-matter is simple and systematic. ‘5. The book serves triple purpose : ‘Text Book—Help Book—Solved University Papers 1 The Point LL. Co-ordinate Geometry. It is that branch of geometry which defines the position of a point in a plane by « pair of algebraic numbers. Itis also called Algebraic Geometry or Analytical Geometry. 1.2, Rectangular Axes and Origin, Let X'OX and Y'OY be two perpendicular straight lines intersecting at the point ©. Then Y @ X'OX is called the axis of ¥ or the x-axis. (ii) YOY is called the axis of y or the y-axis, (id) Both X'OX and Y'OY taken together, in this very order, are called the rectangular axes or the axes of co-ordinates or the co-ordinate axes or simply the axes. They are called rectangular axes because the angle between them js a right angle. (é») Their point of intersection O is calted the origin. 3. Cartesian Co-ordinates ofa Point. Let X'OX and Y'OY be two perpendicular straight lines gal the point O. Let P be any point in the plane of the axes. From P, draw PM L X'OX. Then y @ OM is called the x-co-ordinate or abscissa ot P and is denoted by x (i) MP is called the y-co-vrdlinate or ordinate ot P and is denoted by y. (ii) The mumbers.xand y are called the Cartesian Rectangular Co-ordinates or sitnply the co-ordinates of P. (iv) The symbol PG, y) is used to denote the point P. In this symbolic representation, the abscissa ix always written first and x" Pla, y) ¥ (Ordinate) (Abscissa) x separated from the ordinate by a. comma. Y Remember: 1. Abscissa is the distance (Le. perpendicular distance) of a point from y-axis, 2. Onlinate is the distance of a point from x-2 3, Abscissa is +ve lo the right of y-axis and —ve to the lefl of y-axis. 4, Ordinate is +ve above 5. Abscissa of any point on y-axis is zero. * 6, Ordinate of any point on 7. Co-ordinates of the origin are (0,0) js and negative below x-axis, axis is zero, 2 GOLDEN CO.ORDINATE GEOMETRY DISTANCE FORMULA Article. 1.3. To find the distance between two points whose co-ordinates are given. Let P(x, ¥4) and Q(x, y) be the given points. Draw PL and QM 1 OX and PR LOM. y ‘Then PR=LM=0M-OL Ole Ye) Br=7 RQ=MQ-MR =MQ- LP =n Int. £4, APOR PQ? = PR? + RQ? [Pythagoras Theorem] = @ Hy +O2-9 PO= Vian) + O:-¥)" Remember. Distance between two points (r), )1) and (¢, jx) = Von —n + 02 -nF =V (Difference of abscissae)' + (Difference of ordinates)” Cor. Distance of a point (x, y) from the origin (0,0) =V (r= OF + 6-07 = VE + Example 1, Find the distance benween the following pairs of points = () (am?,2.am;) ; (am;?,2am,) (i) @cosa, asin) ; (acos, asin B). Sol. (i) Reqd. distance = V(am,? — am;)* + (2am, -2am,)* = Var ma + aan my = a Vom mp) + a = 0 Voom, em) (my my Alm = mg) = a (omy — m3) Vm, tae (assuming m, > ms) Cif) Reqd. distance = V(a cos a = a cos f)* + (a sin a= a sin BY = Va" (cosa ~ cos BY + @ (sin a — sin BY? = a V(cos* or sin’ @) + (cos fi + sin? B) - 2 (cas ct cos. fi + since sin B) savi+l—2cos(a—p) =a V2[1 —cos (a -P a2. 2sint BSP e2asin sh * (assuming ct > B) BH Note 1. When three points are given and itis required to prove that : (A) they form an isosceles triangle, show that two of its sides are equal, (i they form an equilateral triangle, show that its three sides are eqaal. i) they form a t. angled triangle, show that the sum of the squares of two sides is equal to the square ofthe th side. are collinear, show that the sum of the distances between two point pairs is equal to the distance between, the third point pair. ‘THE POINT 3 E@FNote 2. When four points are given and it is required to prove that : (i) they form a square, show that all sides are equal and diagonals are equal. (ii) they form a rhombus, show that all sides are equal and diagonals are unequal. (iid) they form a rectangle, show that the opposite sides are equal and diagonals are also equal, (iv) they form a parallelogram, show that the opposite sides are equal. Example 2. Show chat he points (- 2, 3), (1, 2) and (7, 0) are collinear. [Note. “Points are collinear’ means points lie on the same straight line.] Sol. Let A 2,3), B(1, 2), C(7, 0) be the given points, AB =V(-2-1)74 3-2) =VOFT =vi0. BC =V(i- 7) + (2-0) =V3045 = Va = VaxT0 = 2V10 AC=V(-2- 77 + (3 - OF =V8149 = VW =v9x10 =3 VIO xo that AB +BC=Vi0 + 2V10 =3V10 = AC The points A, B, Care collinear ie. tic on the same straight tine. Example 3. Show thai the points (5, 1), (1, ~ 1) and (11, 4) lie on a straight line. Sol. Please try yourself. Example 4. Show that the points (0, - 1), (2, 1), (0, 3) and (—2, 1) are the corners of a square. Sal. Let A(0, ~ 1), B(2, 1), C(O, 3), D(- 2, 1) be the given points. ABeV(0-24(-1- 1 =Vie4 ove BC =V(2- 0)? +(1-3F =vae4=V8 CD =V(04 27 +3 - 1) =V4eF =v DA=V(-2-0) + (1+ 1)? = Ved =v8. ‘Thus the four sides AB, BC, CD, DA are equal. Hence ABCD is either a square of arhombus. In ‘4 square the two diagonals are equal and in a rhombus they are unequal. Let us, therefore, calculate the diagonals. AC#V0+(-1-3) =Wrl6=4 BD=V(242)/ +(1- 1) =Vi6+0 24 « AC=BD Hence ABCD isa square, Example 5. Show that the four points (7, 3), (3, 0), (0, ~ 4) and (4, ~ 1) are the vertices of a rhombus. Sol. Let A(7, 3), BG, 0), C(O, — 4), D(4, - 1) be the given points. AB=V(Q-3)'+(3-0y =VI69 =5 BC=VG—0F +(044) VIF =5 eD=V0- 44-44 IP =V1+o = 5 DAs VG- 7 +(- 1-3) = V9F16 =5 AB=BC=CD=DA. => ABCD iscithera square or « rhombus, In a square the two diagonals are equal and in a rhombus they are unequal. Let us, therefore, calculate the diagonals. 4 GOLDEN CO-ORDINATE GEOMETRY ACaV(7-07 +B 4 4p =VAT4 aD = VOR BD =VG—4) +041) =VI4T = vd ‘Thus AC» BD Hence ABCD isa rhombus. Example 6. Show that the points (3, 2), (17, 8), (8, 12) and (0, 6) are the vertices of a rectangle. Sol. Let A(3, 2), B(11, 8), C(8, 12) and D(O, 6) be the given paints. AB=V@G- 11)" + (2-8)? =V64436 =VI00 = 10 Be =V(l1 8+ (8-12p = VOs 16 =v = 5 CD =V(8- OF + (12 -6)° =VeFe36 =VI00 = 10 DA=V(0- 3)? +(6- 2p =V94 16 = V5 =5 AB=CDand BC=DA => The opposite sides of quadrilateral ABCD are equal. Abo. ACHVG- 8) + (2-12) =V354 100 = VIS BD =V(ii- OF + (8-6F = Vidi +4 = VIS Diagonals AC and BD are also equal. Hence ABCD is a rectangle. Example 7. Show thatthe points (~ 1,0), (0,3), (2,3) and (0,0) are the vertices ofa parallelogram. Sol. Let A(— 1, 0), B(O, 3), C(1, 3) and D(@, 0) be the given points AB=V(- 1-0) + 0-3) =VT49 =Vi0_ BC =V(0- 1)? +(-3F =vIv0=1 cD=V(1- 0) + (3-07 =vIF5= Vid DAsV(0+1)+0 sViF0=1 ” AB=CD and BC= DA => The opposite sides of quadrilateral ABCD are equal. Hence ABCD isa parallelogram. [Note. For another method see examples with Section Formula] Example 8. Show that the points (2a, 4a), (2a, 6a) and (2a +V¥a, Sa) are the vertices of an equilateral triangle. Sol. Let A(2a, 4a), B(2a, 6a) ; C(2a + V5a, Sa) be the given points. AB=V@a- 2a)" + (4a 6ay = VO44e* = 22 BC =¥ (2a ~ 2a - ¥3a)" + (60 - Say = V3a" +a? = 2a CA=VQa4 5a = 20) + (Sa~ 4a)? = V30 4 a = 2a AB=BC=CA, Hence ABC is an equilateral triangle. Example 9. Two points (0, 0), (3, V3) form with another point an equilateral triangle. Find that point. ‘THE POINT 5 Sol. Let A(O, 0) ; BG, V3) be the given points and C(x, y) the third point so that ABC is an equilateral triangle. o AB=BC=CA or AB*= BC’=CA? (0-3F + O-V3Y =G —x + (V5 - y= @—07 + (y-07 or 12227 +y?- bx -W3y+ 12=P+y fi) From the first and third members of (i, we get etyai2 euii) From second and third members of (i), we get P+ y?— Gr -W3y + 12a + y? or = 6 - Wy +1220 or 2W3y =— Gr + 12= 62-3) or yaz%e =x) = V3(2-x) Putting y = V3(2 x) in (ii), we get P43Q-a% 512 or 47-12 + 12 = 1or dx? 12050 Pa3r=0 or x(r=3)=0 2 x=0,3 V3(2 -x)= V3Q-0) = v3 VF(2 -2) = 52-3) =-V3. Hence the third point is (0, 2V3) or G,-V3). Example 10. An equilateral triangle has one vertex at the point (3, 4}-and another at (~ 2, 3). Find the co-ordinates of the third vertex, 53, LeV5_ 7¥5v3 Sol. Please try yourself. [ans 7 5 Jor(2.445°)| Example 11. if the straight line joining the points (a, b) and (c, d) subtends ar angle 0 at the origin, ac+bd then cos 8 = la eC +d) Sol. Let A(a, b), B(c, d) be the given points, so that ZAOB = 0 where O(0, 0) is the origin. OA=V(a- 0) + (b- 07 =Vae +b? y OB =V(c-0F +d -OF Vr ale, 4) ABEV(a-c + 0-aP Using cosine formula in AOAB, we have cos 0 = QA'+ OB? - AB? Aca) 20A.0B = FB) + (24 #- (ae? + (b- dP 2Ve+R Ved a@eh ete P + 2- dace b+d-2d) 0(0,0) x Verse) 2ac + 2bd ac + bd NG PCE) Wars be +d) 6 GOLDEN CO-ORDINATE GEOMETRY ac + bd Ve ste +a)” Another form of above question. /f O be the origin and if the co-ordinates of any two points A and B be respectively (x, ys) and (x3 y2), prove that OA. OB cos AOB = xx) + yiy>. Example 12. The vertices of a triangle are A(1, 1), B(4, 5), C(6, 13). Find cos A. Sol. a= BC=V(4-6)°+ (5-13) =V+ 69 = Vax 17 = 2VIT b=CA=V(6- 1)? + (13-1)? =V25+ 144 =V160 = 13 c= ABeV(1-4) 4 (1-5) = V0e 16 = V5 =5 Becta? 169425- 68 _ 126 _ 63 he 2x13%5 130" 65" Example 13. Find the abscissa of points whose ordinate is 4 and which are at a distance of S units from (5, 0). Sol. Let x be the abscissa of the points so that distance between the points (x, 4) and (5, 0) is 5. Vae-sy+(4-07 =5 or Vx?- 10x+41 =5 Squaring, a 10r+ 41225 or x—10r+ 1620 of (x-2r-8}=0 or x=2,8. s+ Abscissae of the points are 2, 8, Example 14, Show that (3, 3) is the centre of the circle passing through the points (4, 6), (0, 4) and (6, 2) What is the radius of the circle ? Sol. The point P(3, 3) will be the centre of the circle passing through the points A(4, 6), BO, 4) and C(6, 2) if P is equidistant from Ad, 6) A, Band Cie. if PA = PB= PC. Now PA=V@-4) + Q- 6) =VT99 =V10 PB =V(3-0) + (3-4) 3V94T = VIO PC=VG-6) + -2) =vo4T =vIO so that PA = PB = PC P isthe centre of the circle passing through A, B, C. Radius of this circle = PA = PB = PC=Vi0. Hence cos O= cos A= Example 15. Show that (1, ~ i) is the centre of the circle which passes through the points (4, 3), (~ 2, 3) and (6,~ 1). Sol. Please try yourself. Example 16. Find the centre of the circle circumscribing the triangle whose vertices are (~ 3, ~ 1), (1, 3), (6, 2) and find its radius. Sol. Let the circumcentre of the ABC be P(/, k). Since the cesitre is equidistant from all points on the circumference. : PAs PB= PC => PA? = PB? = PC* > (ht 3h + + IP = (ht t+ &-3P = (4-6 + K-2F @ ‘THE POINT 7 or or ‘The first rvo members of (i) give, (he 3h + (+ Ps (he 1 4 &-3" 1417 46h + 2k+ 10 = 1? +B 4 2h 6k + 10 4h + 8k=Oorh+2k=0 haa2k fii) ‘The last two members of (i) give, (ht 1° + -3) = (h-6) + (k- 2) A(-3, = ‘C6, 2) or H+ B+ 2h—6k + 10 =H +B — 12h 4k + 40 or 14h—2k-30=0 Putting h = 2k, we get =28k=2k- 3020 0 or keml 2 ha? 2. Centre of circumcircle is P(2, — 1). Radius of circumcirele = PA(= PB = PC) = V(2+ 3)’ +(-1+ I)? =5. Example 17. Determine the centre of the circle on which the points (I, 7), (7, ~ 1), (5, 3) lie. Whar is the radius of the circle? Sol. Please try yourself. [Ans. (- 8, - 6), 5VT0] Example 18.A line AB is of length 5. A is the point (2, - 3). Ifthe abscissa of the point B is S, prove that the ordinate of B is 1 or ~ 7. triangle. Sol. Let the ordinate of B be y so that B is the point (5, y). Also Ais the point (2, — 3) and AB = 5 “ ViQ=5P+(@3=yF =5 or VO+OeGay a5 Squaring 18+ 6y+y7=25 or y?+6y-7=0 or (V+ T-1)=0 ” ys=Torl Hence the ordinate of B is 1 or—7. Example 19 Show that te poins| L- 3} ( -3,- 5) [-+ =~) retievericesofa righ ansed sou ueta( 1-3 , 37 F}mic -4,-3 tome given pons. 2 2 ap aea?e(-3+3) =164+4=20 = 4 (2,3 = BC? = (-344y +(-3+4) =1+4=5 Ze y4/-3,3)- = ca? =(- 4-1) +(-3+] =254+0=25 AB + BC? = CA? AABCis right angled at B. Example 20. [f the point (x, y) be equidistant from the points (a + b, b a) and (a - b, a + b), prove shat bx = ay. or GOLDEN CO-ORDINATE GEOMETRY Sol, Let P(y, ¥)y/A(@ + 8 ba) and Blab, a + b) be the given points, PA=PB (giver) «PA? = PB? (c-a—b)’ + (y—b 4a)’ = (xa + by + (y—a bP (x= a—by ~ (a+ bY = (y-a—b) yb +a)? (en a~ b+ x—a4 dYe-a—b—x+a~b)= y—a—b+y—b+ ay—a—b-y+b—a) Jusing a — 5° = (a + bY(a—b) (2e-2a)(-26) = (2y—26)(-20) or dbx + dab =—4ay+ dab or — Abr = —day brsay. Example 2L. Find the co-ordinates of a point whose distance from (3, 5) is 5 units and from (0, 1) is JOunits, or or Sol. Let AG, 5) and BQ, 1) be the given points. Let the co-ordinates of the required point be (; »). PA=5 (given) = -V(e-3) +(V-SP HS = (e-3) + (V-5) = 25 = Sty be Oy +920 a-(1) Also PB=10 (given) = Vie—OF y= 1 =10 > 2eQy—1) 2100 => 4y*~2y~9920 en(2) Subtracting (2) from (1),—6r— Sy +108=0 or Sy=108—6r or y= S4=3# 2 From (1), ~-10( 253) +900 16x? + (2916-324 + 9x)? — 96x — (2160 — 120%) + 144 = 0 25x72 = 300r + 900=0 or x7~12e4+36=0 or (r-6"=0 ~ x=6 yo84=30. Hence the required point is (6, 9). ames Pand Q are two points whose co-ordinates are (ar, 2at), ( = (a, 0). Prove that 2, + is constant for all values oft BP % 1 1 Tea ep (« 4 (or?) “aaa Vache aye 1, —__1 vi-nre 1p. 4 * (1-3 2 Sol. ‘THE POINT 1 1 2 — + Se = by + 4ab= b) a ine 1 ly I ee err (2) -—! 1 1 e 1+? =i. a( +P) (+3) +P) aler) aller) @ Thus x 308 independent of and is, therefore, constant for all values oft, SECTION FORMULAE Article 1.4. To find the co-ordinates of the point whieh divides the join of two given points in a given ratio. @ Internally, (ii) Externally. Sol. (a) Let P(x, y3) and Q(t, ys) be the two given points. Let R divide PQ internally in the ratio mm, ; my. . PR_m ie. let Room Let the co-ordinates of R be (x, ). Draw PL, OM, RN ison OX. Through R, draw AB |] OX meeting LP produced in A. and MQ in B. ‘As APR and BOR are similar. AR PA_ PR i RB BQ (given) Now ARI oo Ws RB im, NM * im ON-OL | m ohm OM- ON * i: 3y-¥ me SP myx— mary = MXQ= MX => (my EMC = My + My ayy + me ” xe my + LA-LP__m Also MQ- MB im, = = Am ya-y om => my=myy=my2—my => (my # My = Myy + MY; my tm my +72 Ao | Remember Hence the co-ordinates of R are . my+my > my +My 10 GOLDEN CO.ORDINATE GEOMETRY Cor. Middle Point Formula. If R is the mid-point of PQ then m; = m, ie. R divides PQ internally in the ratio 1: I Lytlin mem 141 2 Loytlyy yty 141 2 Hence the co-ordinates of the mid-point of the line'joining (zy) and (a, y2) are [A ie, Samet abenisne am aan tes | Remember (ii) Let P(x, 93) and Q(r2, yz) be the two given points. Let R divide PQ externally in the ratio m, : mz ie, tet PRom™ a RQ mm," Let the co-ordinates of R be (x, y). Through R, draw a line ||to OX meeting LP and MQ produced in A and B respectively. As APR and BOR are simil R AR, PAL PR ey RB” QB” RO (given) AR_m Now RB" > 9) = ON=OL im ON=OM * m SP mig iat, = mye — mye, > myxa mys mx.— mx, ° L M NX = (m—mye= mra— mx, ee MIM mm PAL™ — AL lm NR-LP _m Alo OB” my MB-MQ~m =~ NR-MQ™m) yoy ms - =m = my—my,=my—m ym ay — my, = MY — mye > MyM = MY2— MY, =H —My = MYr—My m= Hence the co-ordinates of R are | ™22— 761 Mua" Peay | Remember m= my" mys Ed Remember the Ruleto write down the co-ordinates of the point which divides the join of two given points internally or externally in a given ratio. ‘THE POINT Wn Draw any line and write down the co-ordinates ofthe given ye SN points P and Q atits two extremities. Write down the ratio in which mm POisiivided finemally orexternally] and markartowsasshown. pS IE Gx, 9) be the co-ordinates of the point of division, then ty) wa) (i) For intemal division : spa Mi (abscissa of Q) + mz (abscissa of P) 7m +g 1m, (ondinate of Q) + m3 (ordinate of P) Fm (éi) For external division + x am, (ordinate of Q) ~ mz (ordinate of P) Wy — Fig . Remember. A point which divides the join of two given points internally or externally lies on the line joining them [produced in the case of external division}. Example 1. Find the co-ordinates of the point which divides : (i) the join of (2, 3) and (5, - 3) irternally in the ratio 1 : 2. (i) the join of (3, ~5) and (+4, 6) internally in the ratio 2 : 3. Gii) the join of (2, 1) and (3, 5) externally in the ratio 2: 3. Sol, () Let (x,y) be the co-ordinates of the point of division, _15)+2(2)_ Sea Then re 8 _ Me3)+2(3)_-346 Se? TE Pp Co-ordinates of the point of division are (3, 1). (2,3) (ii) Please try yourself. (iii) Let (x, y) be the co-ordinates of the point of division. Then 27832). 265) ~ 3(1) _ 10 - yo BS OS 7 & a 2 Co-ordinates of the point of division are (0,—7). ay (3.5) Example 2. The line joining the points (3, ~ 1) and (6, 5) is trisected. Find the co-ordinates of the points of trisection. Sol. Let PG, ~ 1), Q(—6, 5) be the given points and A, B the points of trisection. A divides PQ internally in the ratio 1: 2 12 ++ Co-ordinates of A are given by : - - P a 8 Q x= 1CO#20) 26, y MELE D1 @-1) (8.5) 12 GOLDEN CO-ORDINATE GEOMETRY Co-ordinates of A are (0, 1). Bis the mid-point of AQ. Co-ordinates of B are given by Sum of the abscissae of Aand Q _ 0-6 ya Suuieeatndoscel Mam 028.5 Sum of the ordinates of AandQ — 1+5 ye DS ++ Co-ordinates of B are (~ 3,3). Hence the co-ordinates of the points of trisection are (0, 1) and (~3, 3). Example 3, The line joining the points (1, — 2) and (~ 3, 4) is wrisected ; find the co-ordinates of the points of trisection, Sol. Please try yourself. Jans. 4,0)3- 8 2)] Example 4. Prove that the points (- 1, 0), (0, 3), (1, 3), (0, 0) are the vertices of a parallelogram. Sol. Let A(— 1, 0), B(0, 3), C(1, 3), D(0, 0} be the given points. Ifthe diagonals of a quadrilateral bisect each other (ie. have DVO, 0) (1,3) the same mid-point) the quadrilateral is a || gm. ‘d.nek . 1 0+3 3 Mia-pia of ACs Oe )-(03) m , (0+0 3+0 3 MispointoreDis(°$°,252)= (0,3) At= 1, 0) B(O, 3) Since AC and BD haye the same mid-point, ABCD is allgm. Example 5. Show that the lines joining the points (1, -2), (1, 2) and (3, 0), (~ 1, 0) bisect each other. Sol. Please try yourself. Example 6.A is (2, 3) and Bis (x, y). C(3, 4) divides AB in the ratio 1 : 5 internally, Find the values ofrandy. Sol. C divides the join of A(2, 3) and B(x, y) in the ratio 1:5 internally. wast NS a 4 6 Co-ordinates of C are ts (SEP fer ae at 23) (8) ey But co-ordinates of C are given to be (3, 4). o ant. and ve. =e x=8 and y=9, Example 7. In what ratio is the line joining the points (4, 5) and (i, 2) divided by @ the x-axis (i) the y-axis? Find also the co-ordinates of the point of division in each case. Sol. Let the required ratio be k: 1. The co-ordinates of the point which divides the join of (4, 5) and (1, 2) in the ratio & : 1 are k+4 2k+5 | ket’ kt ‘THE POINT B (@ If this point lies on x-axis, its ordinate = 0. hea 5 ke 5 20 or k=-5 the x-axis divides the join of given points externally in the ratio 5:2. a SN kit Putting k = ~3 in the abscissa of the point of divisio... s 3 ked cits 3 kel 2iyy 23 xal1 and y=10 ‘THE POINT 15 Example 16. Find the lengths of the medians of the triangle whose vertices are (3, 5), (5, 3), (7, Te Sol. Let AG, 5), B(5, 3), C(7, 7) be the vertices of AABC. Let D, B, Fe the mid-points of BC, CA, AB respectively, so that A(3, 5) Co-ordinates of Dare (FF 3) ie 65) Co-ordinates of E are (77 Jee, 9 F E Co-ordinates of F are (3: =) ie 4,4) Length of median AD = VG - 6° 4(5- 5} =3 25, 3) Ds cw.) Length of median BE = V(S5 = 5)°+ (3 = 6)" =3 Length of median CF = V(7 — 4) + (7 — 4)? =VO49 = VIB = 3V7. Example 17.IfA(— 1, 3), B(I, — 1) and C(S, 1) are the vertices of triangle ABC, find the length of the median through A. Sol. Please try yourself. [Ans. 5] Example 18. Find the vertices of the triangle the mid-points of whose sides are (3, 1), (5, 6), (~3, 2). Sel, Let AG y1), Bsa ¥2)s Css, ys) be the vertices of the AABC and D(3, 1), E(S, 6), F(—3, 2) the middle points of BC, CA, AB respectively. ‘yD isthe mid-point of BC BERAs oe nines Al) pen) mere => yatyse2 ii) F E(5, 6) + Bis the mid-point of CA 3,2) be BBLS = en =10 oi) 6 D c + Betas = ytyel2 wiv) ee ¥2) 61) (79) “+ Fis the mid-point of AB nen . we = tn -) 18 22 = tye op Adding (), (it) and (9), we have AntmenalO or ytutay=5 envi) Subtracting (i), (i), (v) from (vii), one by one, we get * Adding (ii), (iv) and (dw have 1x2 =-5,x5= 11, Wityety)=18 or yy+y2+ys=9 = (viii) 16 GOLDEN CO-ORDINATE GEOMETRY ‘Subtracting (ji), (jv), (vi) from (vii), one by one, we get WaT ME-B ys =S. Hence the co-ordinates of the vertices are A(— 1,7), B(— 5, —3), C(11, 5). Example 19. Find the co-ordinates of the vertices of the triangle che mid-points of whase sides are 0, 9.6 Yand G0. Sol. Please try yourself. [Ans. (0, 0), (1,0), @, 19) Example 20. The three vertices of a rhombus, taken in order, are (2, - 1), (3, 4) ancl (—2, 3). Find the fourth vertex. Sol, Let A(2,—1), BQ, 4), C(-2, 3) be the three consecutive Dixy) c-2,3) vertices of a rhombus ABCD. Let D be the point (x, y). Let the diagonals intersect at M, then M is the mid-point of AC as well as BD. + Mis the mid-pt. of AC Co-ordinates of M are e, 0,1) AZ-1) BY3, 4) Misthe mid-pt. of BD Co-ordinates of Mare (£22, 4) x43 2 Hence the fourth vertex Dis (-3, -2). =0 and ther = x23 and ys-2 Example 21. Three vertices of a parallelogram are (0, 0), (a, 0) and (b, c). Find the co-ordinates of the fourth vertex. Sol. Hint. Proceeding as in Example 20. Mid-pt oacs(3. 5) (222.2 Ao n-ne BDin( #5 z 4) bo ote Sek = = 52 ima S=$ 2 reb-aandyse ya. yn gt or Sy=yityatys OF ys=3y—y1—-Ya Hence the co-ordinates of third vertex are. (3x — 1 — x2, 3y—y1 —¥- _Fxample 6. Two vertices of a triangle are (6, 4) and (3, 2). Ifthe centraid is (5, 0) find the third vertex. Sol. Please try yourself. Ans. (6, -6)] Example 7. Two vertices ofa triangle are (2. 1) and (5, Zandi cenrois( Bt ) Pi ie cia vertex. Sol. Please try yourself. [Ans. (3, 4)] Example 8, Two vertices of @ triangle are (4, - 6) and (2, - 2) and its centroid -1 ) Find the third vertex, Sol. Please try yourself. [Ans. (2, 5)] Example 9. The vertices of a triangle are (1, 2), (1, — 3), (4, K). Find the values of h and k if the centroid of the triangle - at the point (5, ~ 1). Lehe 4 5 ang 23k. jek. 1. Hint —[— 1 => he lke ‘THE POINT 19 Example 10. The vertices of a triangle are (a, 6), (b, ¢= a} (c, a= b) ; prove that its centroid lies on the x-axis, Sol. If the centroid of the triangle be (x, y), then = = bao) +(e~a) + (a-b) 0 aotthte andy bred (ca) +(e b) =9-0, Since the ordinate of the centroid is zero, it lies on the x-axis. Example 11, The vertices of a triangle are (1, ), (2, 6), (2, - 3)5 ( prove that its centroid cannot lie on the y-axis. (ii) find the condition that the centroid may lie on the x-axis, 142+ a+b 3° 3 (9G will lie on the y-axis if its abscissa is zero, ie, if34c%=0 oF ifc=tv=3 Since both these values of ¢ are imaginary. G cannot lic on the y-axis, Gi) G will lie on the x-axis if its ordinate is zero. ie ifa+b—3=0 whichis the reqd. condition. Ss Incentre. The point of concurrence of the internal bisectors of the angles of a triangle is called its incentre. [IEAD, the bisector of ZA of a AABC meets the opposite side BC in D, then from matriculation geometry BD_ AB DC" AC b BC internally in the ratio of the sides containing the angle. Sol. Centroid of the triangle is, 4 ie. Ddivides B D c Article 1.6. To find the incentre of a triangle whose vertices are) given. Hence show that the internal bisectors of the angles of a triangle are concurrent. Sol. Let A(t, y:); BOtz, ya); Cts, J's) be the vertices of the triangle ABC. Let AD, the internal bisector of 4A meet BC in D, then BD_AB_c DC" AC" b ) fe. Divides BC internally in the ratio c : b. ex tbr ope . eth ’ ctb Let the internal bisector of angle B meet AB in I (x, y) so that I is the incentre of AABC. In AABD, because BI, the bisector of 2B meet AD in] DI_BD_BD li Co-ordinates of D are 1A BA™ c BD Le De“b But [From (0)] 20 GOLDEN CO.ORDINATE GEOMETRY = = = = , a From (ii), i ‘Therefore I divides DA internally in the ratio a ; (b +c) yt bry ar, +(b+e) “The eo-ondinates of |are x2 ———————E+_, tht ey aebee atbte syst bya ant b+) "6 = Oithyat cys , asbec a+bec Hence the co-ordinates of the incentre are where a= BC,b =CA,c= AB. The symmetry of these results shows that the biscctors of the angles B and C mect in a point whose co-ondinates are the same as those of L. Thus I lies on all the three bisectors. Hence the biscctors of the angles of a triangle are concurrent, Example 1. Show that the incentre of the triangle whose vertices are (~3, ~2), (5, 0), (+, 3) isthe point (5, 1). . Sol, Let A(—3, ~2) ; B(S, 0) ; C(- 4, 42) be the vertices of the ABC, then no Re VEE VERS c=ABsV(-3 - 5) + (-2-0) =v6de4 a VON = Vax IT = TT. IC (4, ») be the co-ordinates of the incentre, then | Remember ax; thr tery ays + bya toys atbte tht 5 5 1 LEODATO +A g4g+2 pat byt ens S097 (-2)4 S077 (0) +2077 _32+F +2) atte SVT + $VIT + 2V7T S442 Hence the co-ordinates of the incentre are G sD =1 ‘THE POINT 2 Example 2. Find the co-ordinates of the incentre of the triangle whose vertices are (- 36, 7), (20, 7) and (0, ~8). Sol. Please try yourself. [Ans. - 1,0)] Example 3, Find the co-ordinates of the centre of the circle inscribed in the triangle whose angular points are (4, — 2), (- 2, #) and (5, 5). Sol. Please try yourself. (Ans. (& 59] Example 4. Find the co-ordinates of the incentre of the triangle whose vertices are (2, - 2), (8, - 2) and (8, 6). Sol. Please try yourself. ans 91 Example 5.if(0, 4), (2.3) and (2 ,0) be the middle points of the sides of a triangle, find the co-ordinates of ies incenare. Sol. Let D (0,2), EG, 9, FG, 0) be the mid-points of the A sides BC, CA, AB respectively of AABC. Let the co-ordinates of A, B, C be (21,91), (2:2 (X93) respectively. F E(%, A) Dis the mid-pt. of BC (4,0) (2 a) : (° 5) 2° 2 °2 ats = ayo and (0.%) nems0 .@ yytys=l --Aii) Bis the mid-pt. of CA watt Yt) (11) eH ll yg Bed 2° 2 2 2 72 2 2 ” yeaa adit) ystyy= 1 -iv) Fis the mid-pt. of AB + yitya)_ (1 | HY vite * ( 2° 2 ) (3 °) 2 a xptagel (i) yt ped slviy Adaing (©, Gi) and (¥) 264 +. 4.45) 32 xyexpeas= 1 vii) Subsrating() (ii) and (from (i), one by one x21, 20, x20 Aebing (2, () a0 7) 203 492499) =2 ytytys=1 -Avitiy Subtracting (i), (iv) and (v2) from (vi n=O, Co-ordinates of A, B,C are (1,0), ©. (0) and (, 1) respectively. 2 GOLDEN CO-ORDINATE GEOMETRY. To find the incentre of AABC a=BC=V0+Ts1, b=CAsvi¢1sv2, c2ABsvIt0=1. If] &, y)is the incenire of ABC, then yo A +VAT +100) ax; + bx + exs © V4v247 athe --.« 4 Rave“ 2-V2 4-27 2 VE 100) + V3(0) + 1() | + bya + oye yew Tevet "240d atbte =1 “5 (as forx) js 1 1 Hence the incentre of AABC i ~aet-74)- Noteon Trapezium. A trapeziumis aquadrilaternlonlytwoof —_y, whose opposite sides are parallel, ‘Area of a trapezium = lsum of parallel sides) x height. Thus, if ‘Gey 7) Blan) as to pot and AL BM aoa from A on B on 1X, then ALMB is a trapezium. AL and BM being its parallel sides and [hi liedatanes ttwece herpes sides or the height of the trapeziam ‘Area ofthe trapezium ALMB. = + (sum of parallel sides) x height 1 = 3 (AL+ BM)LM =} 61+) (OM-OL) = loti (—m) AREA OF A TRIANGLE Article 1.7. To find the area of a triangle whose vertices are given and hence to deduce the condition of collinearity of three points. Sol. Let A (2,94) | B(>, Yo) $C (ts..93) be the vertices of the AABC. Draw AL, BM, CN Ls on-axis meeting it at L, M, N respectively. Then SABC = Trap. ALNC + Trap. CNMB ~ Trap. ALMB Y Cle, ¥9) = 5 (AL+CN) LN + (CN + BM) NM -} (AL + BM) LM. (AL.+ CN)(ON -OL)+ 1 (CN+ BM) (OM -ON) ~ AL + BM)(OM - OL) 1 #39) (am) + Hs +) 2-9) -F 01+) 2-H) = Heron —y) + Gays xan) + Gavi —ad] 2 z ‘THE POINT 23 ‘To deduce the condition of collinearity of three points. “The points A(x, 91) ; Bsa ¥3) 3 (és, 73) will be collinear (ie, will Hic on a straight line) ifthe arca of the triangle, assumed to be formed by joining them is zero. ie, Hepa and+ esa) + bey) = 9 A A or Gn —22)1) + Gays — 292) + Ga) = 0 Sign of an sirea. An arca will be considered to be ++ve, of in going round the boundary, it always lies to the 8 ce c left ie. if the order of description of the boundary curve is anticlockwise, It will be regarded — ve otherwise. An Important Note. To write the arca of a triangle : 4 vi 1. Write the co-ordinates of the vertices taken in order in two columns. At the end, repeat the co-ordinates of the first vertex. 2. Mark the arrow-heads as indicated. Each arrow-head shows the product. a ve 3. The sign of the product remains the same fordownward arrows whileit = 2 changes for an upward arrow. 2 4, Divide the result by 2. a Ms Thus, A=} [tsy2—ay1) + Gy —x02) + (ai —s yh ~< 5. If the area is given, use complete formula for the area of a triangle ie, 4 vn the usual formula with = sign. Hoyo ot 6. Using determjnants, area of triangle = 4] x;y “)xs ys 1 Example 1. Find the area of the triangle formed by the points {— 3, - 5) ; (5, 2) ; (-9, -3). Sol. Required arca 3 -5 =4C92-OC5)+ OC3)-2) < +(-9) -5)-3)-3)] 5 2 = H-6+25-15 + 18445 ~9] 1 < z ‘ = (88 30) = 458) = 29. ~9 3 Example 2, Find the area of the triangle formed by the points (4, 4), (3, -2) and (~3, 16). 5 Sol. Please try yourself. [Ans. 27] Example 3. If the angular points of a triangle are (at;’, 2at,), (at;', 2atz) and (at;", 2ats), prove that the area of triangle is a(t) - ty) (t2~ 1) (ty~ 4). Sol, Using determinants, the area of triangle at? at, 1 at;? 2at, 1) Taking out ‘a’ and ‘2a’ common from first and second columns ai? 2aty 1 4 GOLDEN CO-ORDINATE GEOMETRY og Pot estate|? mn 1 operating Ry = Ry, Rs—Ry font iF 4 1 Plate? o-t 0 expanding along third column 0 ween? Goh +H 1 Bo a8 | eine | Dont gon | ae Gti —A Bat? oe! ata 5a? (a~h)lls—-4) G8) &@ @ — ty) (6 —ty) in magnitude, FRemember. Three points are collinear if and only if the area of the triangle formed by them is zero. Example 4. Show that the points (a, b + c) ; (b,c + a); (c, @ + b) lie on asiraight line, “ Sol. Area of the triangle formed by the given points = blac+a?—b?—be +ab+ bc? —ac + be +" —a? ~ab] bs 1 =0 3 >< c ath Hence the three points are collinear. Example §. Prove thar the three points (~ 5, 1), (5, 5) and (10, 7) lie on a straight Tine. 5 +e Sol, Please try yourself. Exam ple 6. Prove that the points (~1, 1), (5, 7)and(8, 10) are collinear. Sol. Please try yourself. Example 7. Find the value of kin order that the points (k, 1) (5, 5) (10, 7) " k 1 may be collinear. Sol. The points will be collinear if the area of the triangle formed by them 5 zer0, ise if 5 4 (Sk-5+35-504 10-78) =0 L >< “ 2 or if =%-10=0 or if k=-5. 10 7 Example 8. Determine k in order that the points (2, ~ 1), (~ 3, 4) and tk, 5) may be collinear. k 1 Sob. Please iry yourself. [A bre Example 9. Find the area of the triangle formed by the points (p + 1, 1) (2p + I, 3), (2p + 2, 2p) and show that these points are collinear ifp = 2or—1. Sol. Reqd.arca, = L[3p +3 -2p-14 4p" + 2p - Gp -64 2p + 2-2? — 2] =! Qp?-3p- =} 3p-2) ‘THE POINT is zero. ie. or he. xk. (0, 6), then ath 25 The points will be collinear when area of the triangle formed by them pet i Lag? = if 4 @p?~3p-2)=0 spat it Qp+Ip-2)=0 1 \ =! 2 if s-boor 2 apa Example 10. Show that if (x, y) is any point on the fine joini:x (a, 0) ; Sol. Let P(x, y) ; Aa, 0) ; B(O, b) be the given points. Since P(¢, y) lies on the line AB, therefore area of APAB = 0 1 = = 2 f-ay + ab— bx] =0 => -ay+ab—br=0 > br + ay=ab Dividing both sides by ab, x,t. ate Example 11. Prove that the points (a, 0), (0, b) and (1, 1) are collinear if ty Sol. Please try yourself. Example 12./fthe points (x, y), (1, 2), (3, -4) are collinear, prove thatx +y + 1 = 0. Sol. Please try yourself. Example 13. Find the area of the triangle with vertices (x, y), (x, y*) and (x', y’). Hence find the condition when these three points are collinear. ie, Sol. Reqd. area = ip rye ery] * y why porta? Aye 2 ¥3 Xk =tnl0-9) t97-2)-0+90-9)] , y =ingeniity-9+n) 2 X, = Ly Qn -r-y ty] xx = F970 -911-9)-¥ 1-9] =} 0-901 -2)0-y). ‘The three points will be collinear if the arca of the triangle formed by them is ze70, i Fy = aL -y) = 0 it x20 of y=O of e=y of ¥=1 oF y= Example 14, /fa, b, ¢ are different from each other, prove that the points (a, ), (b, I), (c, C)can never be collinear. GOLDEN CO-ORDINATE GEOMETRY Sol. Please try yourself. [Hint, Area of the triangle = 3 (b— c)(e - a)(a -b) Such a, &, c are different from each other, arca can never be ze10. + The points can never be collincar.] Example 15. If the centroid of « triangle is (1, 4) and two of its vertices are (4, ~ 3) and (~ 9, 7), find the area of the triangle. Sols Let the third vertex of the wianelebe Gay) as 347 4ys The co-ordinates of the centroid are | “= **8 == * 148 | Centroid Formula 5 (= 34). But they are given to be (1, 4) Sem. dey Per and Stag = HStty=3 and 4tyy=12 + 458 and 528 The vertices of the triangle are (4,3), (-9, 7) and (8, 8). Now find its area yourself. [Ans 914) Example 16. //A, B, Care the points (- 1, 5) ; (3, 1) ; (5, 7) respectively and D, E, F are the middle Points of BC, CA, AB respectively, prove that AABC = 4 ADEF. Sal. D, E, F being the mid-points of BC, CA, AB respectively, : 3+5 1+7 Co-ordinates of D are GAS ttt lie 4,4) F E Co-ordinates of E are (° t 1 5 SV ie. @,6) Al=1, 5) Co-ordinates of F are ( = ‘THE POINT 27 [26-8 +6-6+4~ 12]= 48) =4 AABC = Ad DEF. Example 17. The co-ordinates of A, B,C are (6, 3) ; APBC _x+y- Show that aac 7 Sol. For APBC For AABC (— 3, 5); 4, ~ 2) respectively and P is a point (x, y). : } 2 j -2 >< , aM, ppc _ s[5t+3y+6-20+4y+2%] Tet Tyn 14 _rey-2 ABC” TY30+946-204 12+12] 49 7 Example 18. The co-ordinates of A, B, C, Dare (6, 3) ; (= 3, 5) ; (4,2) and (x, 3x) respectively and ADBC 1 ind, AABC 2°77 Sol. For ADBC For AABC Sc" -3 5 i.< 2 Ss x ax AbBC _1 AABC 2 f[Sr4 9x +6=20+12e+2c] 4 $[304+9+6- 20412412] 2 . we M21 7 72 = = well Hence xe ray GOLDEN CO-ORDINATE GEOMETRY Example 19. A, B are the points (3, 4) and (5, ~2). Find the point P such that PA = PB and APAB = 10. Sol. Let the co-ordinates of P be (x, ¥) PA=PB .. PA*= PB? => (r-3)' + (y-4) SP + (+2) = w+ y?— Ge By + 25 = x7 + y?- 10 + dy $29 > 4r-y-4=0 ie x-3y-120 (i) APAB = 10 [Note."* area is given, use complete formula forthe area of a triangle Ze. the usual formula with = sign] 24 [4r-3y- 6-204 Sy+ 2r]= 10 or Gx + 2y-26=220 ie. 6x + 2y-46=0 or 6x + 2y-6=0 ie, 3r+y-23=0 or 3r+y-320 Ta solve (i) and (ii) Multiplying (fi) by 3 and adding to (i) . 10x-70 = 0 bh oxsTye2 To solve (i) and (itip Multiplying (fi) by 3 and adding to @) 10e-10=0 2 k= 1y20, Hence the co-ordinates of P are (7, 2) or(1, 0). Exaruple 20. Jf A(2, 2) ; B(6, 6) are two points, find P so that PA = PB and APAB = 4. Sol. Please try yourself. [Ans.G, 5) oF (5, 3)] Article 1.8, To find the area of a quadrilateral whose vertices are (% y), ¥ = 1, 2, 3, 4 taken in order. Let AQ, 1), BGtz, ¥2), Clta, y3) and D(ta, y4) be the vertices of the quadrilateral ABCD, Join AC. Then Area of quad, ABCD = AABC + AACD Dia vad i. Ys) Als, Ys) Bea, Yo) THE POINT 2» For AABC For AACD ¥ zo eo = ; = z x x = = v1 = 5 bea 2s +922 tt ted +S De a +a ts +91 2D] = Flew 291) + Ges —ay2) + (eye —209) + Gai —7Y0)] Note: The rule for writing down the area of a quadrilateral is the same as that of FL an aatriangle. >< ‘Thus, area of quadrilateral with vertices (x, y,),7= 1,2,3,4i8 * v2 Liaw -aa) + (asa) + ee 20) + (ea - Vd] 1 won Example 1. The co-ordinates of the angular points of a quadrilateral, taken in order, are (I, 1), (3, 9), (5, - 2) and (4, ~ 7). Find its area, X Sol, Reqd. area * a =L[4-3- 6-20-35 + 84447] L 1 2 1 4 soo = 323-64) =-3E 2 2 4 Gin magnitude), 315 oS 2 -4 . 4 7 Example 2./f the area of the quadrilateral whose angular points, taken os, in order, are (1, 2), (~5, 6), (7, ~4), (ky = 2) be zero, find the value of k 1 2 Sol. Arca of quadrilateral =0 (giveny - a 6 c. £(6410420- 42-144 dk + 2k+2]=0 1 yoo 2 2 or &-18=0 t >< 2 ke3. >< 1 2 1.9. Choice of axes A proper choice of the co-ordinate axes simplifies the algebraic calculations invalved in a probleny. For the proper choice of axes, we follow the rules given below: 30 GOLDEN COORDINATE GEOMETRY Rule L Whenever two perpendicular lines are given in a problem, take these lines as the axes, x A(0,0) B{a, 0) A(Q,0) 4 Bla, O) Rule I, Whenevertwo fixed points are given ina problem, take the mid-pointof theirjoin asthe origin, the join itself as the x-axis and the line through the mid-point and perpendicular to the join as the y-axis, Thus, when two fixed points A and B are given, take O, the mid-point of AB as the origin, OB as the x-axis and OY, the line through O perpendicular to AB as the y-axis. If AB = 2a, then Bis (a, 0) and A is (—a, 0). Y Y, th, k) A o 8 x A oO. 8 (2,0) 40.0) (2.0) (a0) (0.0) (a, 0) Rok Role IIL Rule IHL Ina problem, relating to a triangle, take the mid-point of the base as the origin, the base itself as the x-axis and the line through the mid-point and perpendicular to the base as the y-axis. Example 1. Prove shat in a right angled triangle, the Y middle point of the hypotenuse is equidistant from the three angular point. c Sol. Let ABC be a right angled triangle with right angle (©: 2) at A. Take AB and AC as the axes of x and y respectively. Let AB = 2a and AC= 2b, then the co-ordinates of A,B, C are (0,0), Mea, b) (2a, 0), (0, 2b) respectively. 2b Let M be the mid-point of BC, the hypotenuse. Then its co-ordinates are, 0,0) 2a Bl@a,o) * ‘THE POINT 31 Now MA=V(a-0)'+ (6-07 2 Varo MB=V(a - 2a)" + (6-0) =Vars BF MC=V(a - 0)? + (6- 2b)' = Var +b? so that MA= MB = MC. Hence M is equidistant from A, B, C. Example 2. Jn any triangle ABC, prove that AB? + AC’ = 2 (AD? + DC’) where D is the middle point of BC. Soi. Take Das the origin, DC as the x-axis and DY 1. DC as y-axis, Y Let, BC = 2a, then the co-ordinates of B and Care (a, 0) and (a, 0) respectively. Ath, Ky [Remember : Ordinate of any point on x-axis is zero.] Let the co-ordinates of A be (fy 2), then | ABA+ AC = [(i +a)? + P] + [@—a)? +P] =2(K? +P 4°) ) 2(AD? + DC?) = 214? + 2 + a2] i) og D © x From (i) and (ii), we have a0 = @,0) (a,0) AB’ + AC = 2(AD? + DC’). Example 3. ABCD is a rectangle and P is any point in its plane, prove that PA? + PC = PB + PD? Spl. Take AB and AD as axes of x and y respectively. Let AB = a, AD = 6 so that the co-ordinates of the various vertices are as shown in the figure. Let P(i, &) be any point in its plane. (0-2) c PA?+ PC? = (12 +P) + [(ft—a)? + (K=5)"] (a,b) UP + 2 +a? + bP - 2ah —2bk b PB? + PD?=|(h—a)’ + A] + [f+ (k-6))] b = UP + 2 +a? + b= 2ah ~2bk 1x PA? + PC? = PB?4 PD®. 0,0) @ B(a, 0) Exam ple 4, Show that the area of a triangle is four times the area of the triangle formed by joining the middle poinas of the sides of the sriangle, Sol. Let D be the mid-point of the base BC of a A ABC. ¥ Take BC as x-axis, D as origin and a linc through D 1 BC as Alb, k) yraxis. Let BC = 22 so that BD = DC =a ‘Co-ordinates of B ate (~a, 0) E and co-ordinates of C are (a, 0) Let co-ordinates of A be (h &), then hea k co-ordinates of E, the mid-poi = 8 D Cc co-ordinates of E, midpoint of CA ae 5 4) ce wen oo} 2 GOLDEN CO-ORDINATE GEOMETRY co-ontinates of F, the mid-point of AB are (*s* For AABC For ADEF i yy AABC = } [ak + ak] =.ak hea\k _(h-a\k apera3|(*5*)$-(45°)3 Elisa-h+e) [2 vie Hence: ABC = 4 ADEF. Exaruple 5. Prove that three times the sum of the squares on the sides of a triangle is equal to four times the sum of the squares on the medians, Sol. Let D be the mid-point of base BC of a AABC. Take BC as.x-axis, D as origin and a line through D BC as peaxis. Let BC = 4a 50 that co-ordinates of B are (- 2a, 0) and those of C are (2a, 0). Let the co-ordinates A be (2h, 2k). Co-ordinates of D are (0, 0). Co-ordinates. of E, the mid-point of CA are (™ EY ie tak oD c x {0, 0) (2a, 0) 2 72 Co-ordinates of F, the mid-point of AB are ‘Three times the sum of the squares on the sides (BC + CA? + AB?) [(2a + 2a) +0 + (2a — 2h)" + (02k)? + (2h + 2a)" + 2k —0)'] = 3 [L6a" + 4a” — Rah + 4h? 4 4 + 40? + Sah + da? + 47] = 3(24a + BAP + BEY) = 24 (Ba? +h? +P) Four times the sum of the squares on the medians = 4 (ap? + BE*+ CF) = 4 [(2h— 0)" + 2k—OY + (-3a A) + O-k)* + Ga —Ay + (0-8) =A (AIP + AB + 90" + Gah + HP + + 9a + HP 6ah + P) = 4 (80? + 6? + OP) = 24 Ba? + 1 +P) Hence 3(BC? + CA? + AB?) = 4 (AD? + BE? + CF), ‘THE POINT 33 Example 6. Prove analytically that the line joining the mid points of two sides of a triangle is parallel and half of the third side. Sol. See Figure with Example 5, Since the ordinates of E and F are equal, each =k E and F are equidistant and on the same side of x-axis. = EF|| BC Also EF=V(isa—h+ap+(k—b> = V4e =2a BC=4a Example 7. Prove that the centroid of @ triangle is the same as the centroid of the triangle formed by the middle points of its sides. ¥. ‘Sol. Let D be the mid-point of base BC of a AABC. Take BC asx-axis, D as origin and a line through DL BC as axis, Let BC = 62 so that co-ordinates of B are (- 3a, 0) and e those af Care (3a, 0). Let the co-ordinates of Abe (3/4 3k). A(3h, 3k) [F3Taking the co-ordinates in this manner avoids fractions in the co-ordinates of G.] Co-ordinates of D are (0,0). £ ° o (3 0) x 3hs3a 3k) 1 3%9) . 2 °2) 3h-3a #) Co-ordinates of E, the mid-pt. of CA a( Co-ordinates of F, the mid-pt. of AB we( Fs Centroid of AABC is ( Bese Bie } ie GX 3h+3a 3h-3a 3k , 3k OF O+ > 2 272). ; —F> ie. (i ®. «Thus the centroid of ABC is the same as the centroid of ADEF. Hence the result. Centroid of ADEF is Example 8. ABCD isa rectangle and M, the point of intersection of its diagonals. If P be any point in its plane, prove that PA? + PB? + PC? + PD? = 4PM? + AB? + BC?. Sol. Take M as origin and the lines through M parallel to AB and AD respectively asthe axis of x and y. Let AB = 2a and AD = 2b, then the co-ordinate of the various vertices are as shown. Let P be the point (x,y). PA? + PB? + PC? + PD? = (e+ a) + 4b) +e —a) + (+b) + ea? + (y-byP + tal + Or-by = da? + dy? +407 + ab? 34 GOLDEN CO-ORDINATE GEOMETRY 4PM? + AB? + BC? = d(x — 0)? + (y—0)"] + (2a)? + (26)? = 4r + dy? + 4a? + PA? + PB? + PC" + PD = 4PM"+ AB* + BC". Example 9. [f G be the centroid of a triangle ABC, prove that AB? + BC? + CA? = 3 (GA? + GB? + GC*). Sol. Take ©, the mid-pt. of BC as the origin, BC as.x-axis and a st, Jine through O . BC as y-axis. Let BC = 6a, then y co-ordinates of B and C are (34, 0) and (3a, 0) respectively. Let yA(Sh, 3k) Abe (3h, 3k). Then G, the centroid of AABC is (aoe 3k +040 a) AB? + BC’ + CA? Gh + 3a)? + (Bk -OF + (6a)? + Gh-3a)? + Bk-o7 18? + 187 + Sda? = 18 07 + + 3a) 8 ° cx 3 (GA" + BG? + GC) (32, 0) (3a, 0) =3[(h—3hP + 3k) + (he Bay + P+ 30) +P] 3 (6h? + 6H + 1807) = 18 (lr +P +30) AB? 4 BC’ + CA? = 3 (GA? + GB? + GC). Example 10. Prove shat the joins of the middle points of the opposite sides of a quadritaterat and the join of the middie points of the diagonals meet in a point and bisect ane another. Sol, Let ABCD be the quadrilateral whose vertices are ACG, ¥1), BGs, ¥2), Ces, ya) and Dix, ya) respectively. Let E, F, G, H be the mid-pts. of AB, BC, CD, DA and L, M be the mid-pts. of AC and BD respectively. Coontinaes of Bar (5, 3). ) ie (hb. 2 Co-ordinate of mid-pt. of EG are Corinne of Gare( 5%, 257), Mites tate Ntya Yat ye 2 2 2 2 2 2 ie, (tReet newton A € 8 i, (a, 7 : Similarly the mid-pts, of FH and LM have the same co-ordinates. Hence the results. 1.10. Locus and Its Equation We know that the sct of points equidistant from two given points lic on the right bisector of the line joining the given points. All points ata given distance from a fixed point lic on a circle, ‘THE POINT 35 It follows that a set of points, not taken at random, but obeying some definite condition, may lie on a straight line or on a curve which we may call their path, If (, y) are the co-ordinates of a general point on the path, they may be bound by an analytical equation ie, by an equation involving the variables x and y or only x or only y. Thus. ‘The path traced by a point moving under a given condition (or conditions) is called its locus. ‘The equation satisfied by x and y, the co-ordinates of any polnt on the path is called the equation of the locus. This equation is satisfied by all the points on the locus and by no othet point. Important. To find the equation of a locus, proceed as follows: 1. Select suitable co-ordinate axes, if necessary. 2. Take a point on the locus and suppose that its co-ordinates art (4, y). 3. Properly conceive the given geometrical condition which the above point (x, y) is to satisfy. 4, Express the said condition in an analytical relation in.x and y and simplify. 5. The relation (or equation) is the required equation of the locus, Example 1. Find the equation of the locus of a point which moves so that twice i x-avis is five times its distance from the y-axis. Sol. Let Pz, 9) be any point on the locus. Distance of P from the x-axis = its ordinate = y Distance of P from the y-axis = its abscissa =x. By the given condition 2y=Se or Sx—2y= 0 which is the reqd. equation of tbe locus, Example 2. point moves sa that its distance from (3, 0) is twice its distance from (- 3, 0). Find the equation of its locus. Sol. Let P(e, y) be any point on locus and A(3, 0), B(—3, 0) be the given points. By the given condition PA=2PB = Vee=3F + = OF = 2V (043) = OF Squaring both sides, 7 + y" Gr +92 47 4 74 Gx 4 9) or Bx 43y7 430042720 or 4574104920 which is the required equation of the locus. Example 3.A point moves such that the algebraic sum of its distances from two perpendicular lines is equal to ‘a’, Find the equation of its locus. Sol, Take the two & lines as the axes of co-ordinates, Let P(x, y) be any point on the locus. Distance of P from x-axis = its ordinate = y Distance of P from y-axis = its abscissa = x By the given condition #y =a whieh is the reqd. equation of the locus. Example 4. Find the equation of the locus of a point which moves so as to be at equal distances from the point (a, 0) and the y-axis. Sol. Let P(x, y) be any point on the locus and A (a, 0) be the given point, Distance of P from y-axis = its abscissa =x. By the given condition. PA = distance of P from y-axis, = Vee-a¥ +y" distance from the 36 GOLDEN CO-ORDINATE GEOMETRY ‘Squaring both sides, x? +y’-2ar +a" =x? or y?-2ax¢a7=0 which is the required equation of the locus. Example 5. Find the locus of a point which moves such that its distance from (3, 2) is equal to its distance from the y-axis. Sol. Please try yourself. [Ans. y? ~ Gr ~4y + 13 = 0] Example 6, The ends of the hypotenuse ofa right angled triangle are (0, 6) and (6, 0). Find the locus of the third vertex. Sol. Let A(0, 6), B(6, 0) be the given ends of the hypotenuse and (6, 0) PG y), the third vertex. Since APAB is 1. £d PA? + PB? = AB? > GOP + Y-6f + 6-6) + y-0) = @-67 + (6-077 me t+ 12y 4364.27 +? 12% +36 = 36 +36 > Beagrie tone = +y¥-Gr-6y=0 which isthe required locus of P. Poy) 00.8) Example 7.A point moves such that its distance from the axis of x is half its distance from the origin. Find the equation of its tocus. Sal. Let P(x, y) be any point on locus. Distance of P from x-axis = ordinate of P=y By the given condition, Distance of P from x-axis = } OP where O is the origin (0, 0) ” y=iv(r-0)+(y-oF => aeVery, Squaing4y?=x7+y? = 2-3y=0 which is the reqd. equation of the locus of P. Example 8.A(x;, 9;), B(x y2) are nwo given points. Find the locus of P such that ZAPB = 90°. Sol. Let P be the point (x, y), Inn. £d. APB, Pty) AP? + BP? = AB? Gy + yD T+ e312? + 9-99) = 1-22)" + G1 -ya)? > Paap Dery + y? +P Dyyy eat ee? Der, + y? + py? —2yy x say tay -2eyn ty? +P — By O64, 94 Oa) me De Dy? Dery — Derg 2yy) — 2yyn + Zaye + 2yya =O = (P+ VG + ede 01+ yay + ie ty ye) =O which is the reqd. equation of the locus of P. o THE POINT 37 Example 9. If the join of the points (2, 3) and (~ 2, 5) subtends a right angle at the point P, find the locus of P. : Sol. Please try yourself. [Ans. 27 +? — x —8y + 13 0] Example 10. A(a, 0) and B(- a, 0) are two given points. Find the locus of a point C if the angle ABC isa right angle. Sol. Please try yourself. [Ans.x? +)? =a?) Example 11. Find the equation of the right bisector of the line joining (3, - 4) and (4, 5). Sol. Let P(x, y) be any point on the right bisector. Let A{—3, —4), B(4, 5) be the two given paints. Since any point on the right bisector of the line joining the two pots is equidistant from. the two points, oo PA=PB. = PA? = PB? = (e439 + (y +4)? = 4 + (5) we Paya Gr By +25 x8 +)? fr 10y +l 1dr + 18 16 = Dor 7x + 9y—-8=0 which is the required equation of the locus of P i.e. of the right bisector of AB, Example 12. Find the equation of the locus of a poine which moves so that its distance from the poirt (ak, 0) is k times its distance from te poin(f 0) c[eee | Sol. Let P(x, y) be any point on locus and A(ak, 0), B ( f.0 Je two given points. According to the given condition, we have PA=k.PB > Viewers 9-07 = [ i} +o of Squaring, Payee rdP ek ( ey Bre 8) = 4+y abs Pe Be + By Dake + a? = (1- Ppt 4 (1- By? - BP) = Dividing throughout by (1 —), we have 27 +) which is the required equation of the locus of P. Example 13. Find the locus of a point whose distance front the origin is to its distance from (~ 2-3) as 5:7. Sol, Let PG y) be any point on locus and A(- 2, ~3) be the given point. According to the given condition, we have ice of P from origin (0,0) _ S PA 7 5 7 - try Veer 2)? + +3)" 38 GOLDEN CO-ORDINATE GEOMETRY Squaring, tt wey odes Gy 413 49 = 49x? + 49y? = 25x? + 25y7 + 100x + 150y + 325 = 24x? + 24y7— 100r - 150y -325 =0 the required locus of P, Example 14, Find the lacus of a poins P which moves so that the sun of the squares of its distances from A(3, 0) and B(- 3, 0) its four times the distance between A and B. Sol. Let P be the point (r, »). By the given condition PA? + PB? = 4AB which gives? +y?=3, the reqd. locus of P. Example 15. A(1, 2) and B(3, 4) are to fixed points. Find the locus of P if 3PA? + SPB? = 4. Sal, Please try yourself. [Ans, 22 + 2)? - 9x - 13y +342 0) Example 16.A(1, 1) and B(-2, 3) are two fixed points. Find the locus of a point P which moves so that area of & PAB = 9. Sol. Let P be the point (x, y) ‘=< Arca of APAB = 9 1 L y . 1 se elr—y4342-2y-ar] 29 ; aos, > ~2e-3y 452218 >< => Qet3y413=0 or W+3y-35=0 x »| which is the reqd. locus of P. Remember. If in a problem two points are given, take the middle point of the line joining them as. origin and the line joining them as x-axis. Example 17. Find the locus of a point which moves so that the sum of the squares of its distances from two fixed points is constant and = 2c*. Sol. Let A and B be the two fixed points. Take the line joining A and B as.r-axis and O the middle point of AB as origin. Let AB = 2a, so that the co-ordinates of A and B are (~ a, 0) and (a, 0) respectively. Py) + Let P(x, y) be any point on locus. According to the given condition PA? + PB? = 2c* we eta) ty + e—a)t ty 2? > 20 + 2y? 2a? = 2c? A Bx => P4y?= 2d which isthe required locus of P, (9) (2.0) Example 18.A point moves so that the difference of the squares of its distances from two fixed points (2, 0) and (~a, 0) is constant = 212. Find the equation of its locus. Sol, Let P(s, ») be any point on Jocus and A(a, 0), BC a, 0) the two fixed points. By the given condition PA?-PB?=2@ if PA? > PB? PB*- PA? = 2? if PB*> PA? ‘THE POINT 39 or PA? PB? = = 27 or [e@na)?+Y)]- [ee +0) + y']= 2 2P or —dav=22 or Jaret =0 which is the reqd. equation of the locus of P. Example 19. Find the equation of the locus of a point such that the sum of its distances from two fixed points is constant and equal 10 2a. Sol. Let A and B be the two fixed points. Take the line ¥, joining A and B as x-axis and O the mid-point of AB or origin. Let AB = 2c s0 thst the co-ordinates of A and B are (—¢, 0) Phx y) and (c, 0) respectively. Let P(x, y) be any point on the locus. “w Acconding to the given condition, we have PA + PB = 2a a Vero ey +Va-cP ty = 20 Ai) [eatically, we have [e+ cP +y]=[e cf + 7] = dex wi) A ° 6B x Dividing (i) by (-e.0) (e.0) Verey-Ve-oay = it iit) Adding (#) and (ii), we have Woaroey = ta EE ie VeereP ey are xt Rated? ‘Squaring, (e+ fay? aa’ tay tie or Beta dert yet SE p2er or (1-G)for-ee ot aed @ > Dividing throughout by (a? —c*), we get ae ee = t which is the required locus. Example 20. A(ac, 0) and B(- ae, 0) are two fixed points. A variable points P moves such that PA + PB = 2a, Find the locus of P. 2," Sol. Ple olf. ‘Ans. 1 lease sry yourself. Sata ] Example 21. Find in its simplest from the equation of the locus of a poine which moves such thatthe sum ofits distances fram the poiras (1, 0) and (- 1, 0) is 3. Sol. Let Pf, y) be any point on locus and A(1, 0) ; BE 1, 0) be the given points. According to the giveri condition, we have PA+ PB=3 or VGq- 1+ + Vier Psy =3 oo) Identically, we have {19+ I-Ie + 1+ =a ~Ci) 40 GOLDEN CO-ORDINATE GEOMETRY Dividing (i) by, Ver= D497 View Fey? =- waCity Adding (() and (ii), we have Woe -= ‘Squaring, a4 P2041) =94 Ea or 42442 —srs dos 1 ay or aes aees s Ie or 36x? + 36)? = 45 + 16:7 or 20x + 36y?-45=0 which is the required equation of the locus. Example 22. Find the locus of a point"P which moves so that PA + PB = 5 where co-ordinates of A are (2, 0) and those of Bare (- 2, 0). Sol. Please try yourself. [Ans. 367 + 100)? = 225) Example 23. Show that the equation of the locus of points such that the difference of their distances from wo fed points (ae, 0) and (- 06,8) is 2a, 8 "—— As = ds Sol. Let P(r, y) be any point on locus and A(a, 0) ; B(—ae, 0) be the two given points. According to the given condition, we have |PB-PA|=2a {the symbol ~ stands for difference] ae PB-PA=+2a or Veeraey ry -Vr—aey + y = 220 ei) Identically, we have [Gc + ae)? +57] = [(r—aey + y?] = daer wali) Dividing (ii) by @, we have, Vicsaey ty? + V(x aey + y? = 4 2ex Adding (i) and (iii), 2V(r4 ae)? +)? =#2(a + ex) oF Vier aces y = (a+ex) ‘Squaring (eae) + y? = (a+ ex? or Peet Qaer+ yea? + ext + Qaer or Pe@etysat+er or (?-Ie-ysa(e-1) Dividing throughout by a%(e? - 1), we have 2 2 Yt which: . ; FBT Mbieh isthe required locus Example 24.A(1, 2) and B(3, 4) are two fixed poinis. Find she locus of P in each case : PA_2 PA2 : iit) 3A? = Ops7F (i) 2PA + 3PB = 5 Gil) 3PA? + SPR = 4. PAL? apas 2 app? Sol. (i) ppt} 7 3PA=2PB = 9PA’=4PB' Hoe ~ 1° + YW = Ae ~3)7 + 4] or 5x? + Sy? + Gx —4y—55= 0 which is the reqd, Hocus of P, ‘THE POINT 41 (ii) 2PA + 3PB = 5 | - 2V (c= + OF 46-3 = Wa = : a Wik eye ay 5 43 Vi ty B25 BS } so Wat ey? = Dr dy 45 = 5-3V7 + y? = Gr = 8p 4 25 Squaring 4(x7 +? -2v—dy +5) = 25-30V + y? - Gr — By 425 +9(07 +? Gr —8y 4 25) se — Sx? Sy? + 46x + 56y 230 = 30 V2 + yr By 42S Squaring (Sx? + Sy? - 46x — S6y + 230)" = 900(2" + y? — Gx — 8y + 25) which is the reqd. locus of P, (iil) Please try yourself. Pans, 2x7 + 27 — Or — 13y + 34 = 0] Example 25. A(3, 4), B(~ 2, 1) are two fixed points and P moves. Find the locus of P in each case : (0) 4PA? - 3PB = 5 (ii) 2PB = 3PA (iii) 2PB + 3PA = 5. Sal. Please try yourself, [Ans. (i)? + y?-36r -38y + 80 = 0, (i)x7 +? - Ide 16y 4 41 = 0 (iii) 36(2 + y? — 6x = By +25) = (7 + y? - 14 — L6y + 46)") Example 26. A(a, 0) and B(— a, 0) are two fixed points. Find the locus of a point P which moves such that 3PA = 2PB. Sol. Please try yourself. [Ans. 5x? + 5)? - 26ax + Sa? = O} Example 27. A line of length 2 slides with its ends on two perpendicular lines. Find the locus of its mid-point, Sol. Take the two perpendicular lines. as axes of co-ordinate. y, Let AB be the line of length 24. In any position, B let OA=a, OB =b (0, b) sothat from mt. £d AOAB OA? + OB? = AB? M or eb sn 5 or ee AP wo (i) Co-ordinate of A and are (2, 0); (0, b) respectively. The co-ordinates af M, the middie pointof ABarc givenby a Afa, 0) Putting these values of a and b in (i), we get 4tedy=4F or Peyt=P which is the required locus of M, the middle point of AB. Example 28.A stick of tength I rests against the floor and a wail ofa room. If the stick begins to slide on the floor, fird the locus of its middle point, Sol. Please try yourself. [Ans. dr? + dy? = PY 2 The Straight Line Article 2.1. To prove that every equation of the first degree in x andy represents a straight line Sol. The general equation of the first degree inx and y is Ar+By+ C0 (i) Let (x 91) and (2, y2) be any two points on the locus represented by (i). Since the co-ordinates of every point onthe Joes satiny the equation of he locus . Ax, +Byy + C=0 ii) Axy + Byz+C=0 (Gi) Muhiplying (ii) by &(k » — 1) and adding to (fi), we have AGt + kx) + BY, +49) 4 C14 = 0 e+e Baty - . or a( ta )-2(4 bei |t er iv) of ket, bt yn Equation ()showsthatthe point| + — , “727+ | which divides the straight line joining the points (1,92) and (23, ¥3) inthe ratio k 1 lies om the locus represented by (). Now kis an arbitrary constant. Therefore every point on the straight line joining (xs, y,) and (x, y2) lies on the locus represented by (i). Hence Ac + By + C =O represents straight line. Second Method ‘The general equation of the first degree in. and y is Ar+By+C=0 O Let P(x, ¥1)s Obras 72) anid Rérs, v3) be any three points om the locus represented by the equation (i). Since the co-ordinates of any point on locus satisfy the equation of the locus ” An+By+C=0 Ant By tC=0 Aty+ By + CeO Solving (ii) and (iv) by cross-multiplication for A, B,C, we have Bo mr G2 Ese ia EY) ‘Substituting the values of A, B, Cin (#) KlxyQ2~ Ys) + yes —22) + (ays —t2)] =O or p> — 321) + Gays — E02) + Gay: — 1s) =O t- ke0] or Dle2— 21) + (rays 9d + ar —ny)] =O ‘This shows that the area of the triangle formed by P, Q, R, is zero. 42 ‘THE STRAIGHT LINE 43 the points P, Q, R, lic ona straight line. But these are any three points on the locus represented by (@). Hence the equation (i) represents a straight line. Exercise, Show that the locus of the equation lx + my-= 1 is a straight line, Example. Show that the locus of a point equidistant from two points is a straight line. Sol. Letthe line joining the two points A, B (say) be taken as.x-axis and its mid:pt. as the origin. [f the distance between the ¥ ‘two points is 2 a, then their co-ordinates are A(- a, 0} and B(@, 0). Let P(x, y) be any point on locus. By the given condition PA=PB = Viera ty =Vr-a ey? Squaring (x+ a)? +y?=(@-a)+y* a a = ax = 0 (on simpliti ” © x or x= Owhich is the equation of y-axis. (2, 0) Hence the locus is a straight li Article 2.2. To find the equation of a straight line parallel to the x-axis and at a distance b from it. Let AB be a straight line parallel to the x-axis and at a distance 6 from it. Let P(x, ») be any point onthe line. Draw PM 4 x-axis. Then MP = b forall positions of the point P. But MP=y Le is the required equation of line parallel to the x-axis and at a distance b from it. line which is parallel to x-axis does not Cor : If the line co-incides with the x-axis, b = 0 “Hence equation of x-axis is y = 0. Note ; If the line is abave x-axis, b ix +ve. If the line is below x-axis, b is~ve, Article 2.3. To find the equation of a straight line parallel to the y-axis and at a distance a from it. y, 8 Let AB be a straight line parallel to the y-axis and at a distance a from it. Let P(x, y) be any pointon the line. Draw PN y-axis. Then NP =a forall positions of the point P. But NP=x Le] is the required equation of a line parallel to y-axis and ata distance a from it. ‘ 4 GOLDEN CO-ORDINATE GEOMETRY ‘The equation of a line which is parallel to y-axis does not contain y. Cor : If the line coincides with the yeaxis, a = 0. Hence the equation of y-axis, is Note. If the line is to the right of y-axis, a is +ve. If the line is to the left of y-axis, a is— ve. Example 1. The equation of a line paraltel to y-axis and at a distance 5 units (i) above itis y = 5 Gi) below itis y == 5. Example 2. The equation of a line paraliel to y-axis and at a distance 2 units (i) to its right is x = 2 (ii) to its leftisx = ~2. Example 3, Find the equation of the straight line paraltel to x-axis and passing through the point (6, 3). Sol. The equation of any line parallel to x-axis is y=b od) Ifis passes through (6, 3), then : (i becomes which it the required equation of the line, Slope or Gradient ofa line. The tangent of the angle which a line makes with the positive direction of x-axis is called the slope of the gradient of the line. It is generally denoted by m. v4 Y a) ® ° x ‘Thus, if a Line makes an angle 0 with x-axis, then its slope = tan 0, ie, m=tan0 Ita line is parallel tox-axis, 9 = 0 ao stan =0 If a line is parallel to y-xis, itis perpendicular to x-axis so that 0 = 90", m = tan 90° =o, Article 2.4. To find the equation of a straight line passing through the origin and having a given slope m. Let OB be the straight line through the origin O making an angle 0 with the positive direction of x-axis. B Then its slope m = tan 0, Let PG, y) be any point on the line. Draw PM L.1-anis so Pty) that OM = x, MP = y. From the right angled AOMP, we have ‘THE STRAIGHT LINE 45 Example. Find the equation of the line through the origin and having an inclination of 120° with xaxis, Sol. Here O= 120° maton 0 stan 120° = —tan 60° ==V5 . Equation of the line is yeaV5.x ie V3.x+y=0. Article 2.5. Slope intercept Form. To find the equation of a line having a given slope ‘m’ and making a given intercept ‘c’ on the y-axis. Let AB be the given line meeting OY in B so that OB =c, Let © be the inclination of tbe line with x-axis. ” mound Let P(x, y) be »ay point on the line. Draw PM 1 OX and BQ 4. MP, then CQBP= ZXAP=@ [Corresponding 4s] BQ= OM =x and QP = MP-MQ =MP-OBsy=c 2. From right angled ABOP, we have oP Y=€< morye= BQ tan6 or x mory=c= me or yamx+c which is the required equation. Note. ‘c* is + ve or —ve according as the point B lies above or below the x-axis, Cor. 1. If the line passes through the origin, ¢ = 0. Equation of line is y = mr. Cor, 2, If the line is paralle] to.x-anis, its inclination with x-axis is 0. = m=tan0s0 -. Equationofline isy = c. Example 1. Find she equation of the straight line which cuts an intercept + 5 from y-axis and makes an angle 3" with x-axis, Sol. Here <=5, 0=30° mounds tan30 = Je Using slope intercept form, the equation of the line is, yemete ie. yageres. Example 2. Write down the equation of the straight line which makes an angle tan 2 with the x-axis sand cuts off an intercept 5 from the negative direction of y-axis. Sol. Here Osun'2 o WaO = 2 mestanO=2 c=-5 ‘Substituting these values in the slope-intercept form y = mx + c, the required line is y = 2r—5. Example 3. Find the equation of the line which passes through the point (0, 1) and has an inclination 45° with x-axis, 46 GOLDEN CO-ORDINATE GEOMETRY Sol. Let the required equation be y = mx +c ) Here m=tand4s*= 1 @ becomes yerte As the point (0, 1) lieson 2 IsOteorcst from (i) the required line is given. yareh Example 4, Find the equation of a straight line cutting off an intercept-1 fram the y-axis and being equally inclined to the axes, Sol. Here c=-l 0=45° or 135" mound ° or tan 135° 1 45* or tan (180° — 45°) n 45° or —tan 45° =stan4S°=21. Kar(Remember : The slope of a line equally inclined to the axes, is#1) ‘The equations of the lines are yeex-l ly=mete ie, x-y-1=0 or rey41=0, Article 2.6. To reduce the equation Ax + By + C = 010 the slope-intercept form and hence, ta find its slope, Sol. The given equation is Ax + By +C =0 or Bys-Ar-C or ya8. alo which is of the form yemrte A slope m = =F. Hence slope of the line Ax+By+C=0 is co-efficient of y [Remember Example. Find the inclinations of the following lines with x-axis @x-y41=0 (i Vix+y +220. Sol. () = unO=1 + 0245" () = —tan 60° = tan (180° - 60"}=tan 120° +. 0-= 120°. Article 2.7. To find the slope of the lite joining two given points. Let P(xi, 91) ; (ca, 2) be the two given points. Let 0 be the inclination of the line PQ with a-axis so that m = tan 0, ‘THE STRAIGHT LINE 47 From Pand Qdraw PMand QN Jstothe r-axisanddraw PRL QN. Let PQ meet OX in A. ZRPQ = £XAQ=0 (cores. £3) PR =MN=ON-OM =x; -2; RQ=NQ-NR =NQ-MP=y2-¥ From the right angled wiangle POR, we have ROL. yo une ae Eo Gor” Remember : Slope of the line joining the points (<1, y1) O94 “ N and (a, ya) is * | Remember Example (3, 3) ; B(- 3, - 5) and C(- 1, 6) are the vertices of MABC. Find the slopes of the medians, Sol, Let D, E, F be the mid-points of sides BC, CA. AB respectively. Co-ordinates of D are ag.) z3-1 =$46);, (_2t 2° 2 2 Slope of median AD F € = D c 5) 1,8) -5- Slope of median BE = 1, -3-1 -4 8 3-3 3-5 Coordinates of Fae (252,255 ie (0,-1) v4 Slope of median CF = 252) « 7 27 8 Intercepts. Let a straight line AB mect the x-axis in A and the ypaxisin B. Then (@ OAs called the intercept of the straight line on.x-axis. (i OBis called the intercept of the straight line on y-axis, A ‘The intercept on x-axis is positive ifthe point A lies to the right “O x of O and negative if it lies to the left of O. ‘The intercept on y-axis is positive if the point B lics above the point O and negative if it lies below O, (iii) AB is the portion of the line intercepted between the axes. * 48 GOLDEN CO-ORDINATE GEOMETRY Article 2.8. Intercept Form : To find the equation of the line whose intercepts on the axes are given. Let the line meet the x- in A and the y-axis in B. Let OA = a, OB =b. Let P(x, y) be any point on AB. Drw PM 2 OX. From similar triangles MAP and OAB, we have MP_ MA OB” OA or or is the required equation of AB. Remember. The equation of the line whose intercepts on the axes are a and b is, L ys Rermembe fe. wis * Intercept on axis"! ' . Example 1. Express Ax + By + C= 0 in the intercept form. Write down its intercepts. Sol. Ar+By+C=0 or Ar#By=-C Ar, By toe or Ae Phat or Feat B which isof the form = +% = 1 a‘b ent Intercept on ris = c : Intercept on y-axis =~. Remember. To find these intercepts directly : 1. To find the intercept on x-axis, put y = O and find x. 2. To find the intercept on y-axis putx = 0 and find y. “Thus, to find the intercepts made on the axes by the line dx-yt220 Intercept on x-axis = ===) (putting y = 0, 4x +2=0,x=-3 Intercept on y-axis = 2 . [+ puttingx = 0,-y+2=0,y= 2) ‘THE STRAIGHT LINE 49 Example 2. Find the equation of a straight line which passes through the point (1, - 2) and cuts off equal intercepts from the axes. Sol. Let each intercept be ‘a’ Equation of the line is given by Xyter ie xeyra i) “the point (1, ~ 2) lies on it 1-220 of @ from (j), the required equation is x+y =—1. Example 3. Find the equation of a straight line which passes through the point (4, 6) and makes intercepts on the axes equal in magnitude but opposite in sign. ‘Sol. Let the intercepts be a and — a, then the equation of the line is ryt. -ys - Eetat ie x-yea i) the point (4, 6) line on the line, its co-ordinates satisfy equation). -. 46 =aora==2. Required equation x-y=-2ie. x-y+2=0. Example 4, Find the lines through (4, - 3) whick cut the axes so that the intercepts are equal in magnitude. Sol. Intercepts are equal in magnitude. They may or may not be equal in sign. (@ When the intercepts are equal in magnitude as well as in sign, let each be ‘a’. 1 Equation of the line is Zeke ie xtyna 49) the point (4, — 3) lies on the line. 4-350 or ast Required equations = x #y=1. (Gi) When the intercepts are equal in magnitude but opposite in sign. Let the intercepts be a and — 0. Then equation of the line is Ey a*na sl ie xreysa ~@ The point (4,—3) lies on the fine B 4-(3)2a or a=7 4 Required equation x-yeT, Example 5. Find the equation of the straight line which passes through the point (1, - 3) and makes an intercept on the y-axis twice as long as that on the x-axis. [Ans 2r+y 4120) Example 6. A line passes through (— 3, 10) and the sum of its intercepts on the axes is 8. Find its equations. i Sol, Let the intercepts on the. axes be a and 5 so that atb=8 = [Equation of the fine is aepel anil) 50 GOLDEN CO-ORDINATE GEOMETRY “2 the point (3, 10) Hies on the line =3,10 Sept ie 10a-3b=ab sn(iti) From (), be8-a From (iii), 10a~3(8~2) = a(8—a) or 1a~24+3a=8a-a? or a?+Sa-2420 or (a+ 8)(a=3)=0 6 e838 when a=-8, be8-a=8+8=16 when a=3, bs8-a=8-355 so that there are two lines satisfying the given conditions and their equations are Age and jaded ie. 2r-ys-16 and Sx+3y=15 ie. 2e-y+16=0 and Sr #3y=15. Example 7.A straight line passes through the point (i, 1) and the portion ofthe line intercepted between the x-axis and the y-axis is divided at this paint in the ratio 3 : 4. Find the equation of the line. Sol. Let the equation of the line be atte 1 ¥, snl) If it meets x-axis and y-axis in A and B respectively: then OAsa OB=b so that the co-ordinates of A and B are (g, 0) and (0, b) respec- tively. IM divides AB in the ratio 3 : 4, then co-ordinates of M are 3(0)+ 4a 3b+ | ie 344° 344 ” But the co-ordinates of M are given to be (1, 1). 4a 3b pe land = 1. = ast =f 2p =5: Substituting these values in (), the required line is. F+¥ el, ie, Sei ie dx+ dy 27. a3 Example 8. Find the equation to the line which passes through the given point (- 4, 1) and is such That the portion of it between the axes is divided by the point in the ratio 1 : 2. Sol. Please try yourself. [Ans x-2y 4 6=0] Example9.A straight ine passesthrough he point(2, 3) and the portion ofthe line intercepted between the axes is bisected at the point. Find its equation. Sol. Let the equation of the line be ate 1 ~@ ‘THE STRAIGHT LINE 51 If it meets x-axis and y-axis in A and B respectively, then OA=4,0B=b 0 that co-ordinates of A and Bare (a, 0) and (0, 5). ¥ If M is mid-point of AB, then co-ordinates of M are a0 Osb & ( 2° 2 ) $3) (Ob But the co-ordinates of M are given to be (2, 3) . a. bL * $2 and 5=3 > az4, b=6 Substituting these vatues in (i), the required line is. e = lorax + 2y= 12. Example 10, straight line passes through the point (4, 5) such that the portion intercepted by the ares is bisected at that point. Find the equation of the straight line. Sol. Please try yourself. [Ans, Sx + 4y = 40] Example L1.A straight line is such that the portion of it intercepted between the axes is bisected at the point (, 9). Prove tha is equation is 5+ 3 = =i. J. Let the equation of the line be = + ¥= 1 m0) If it meets x-axis and y-axis in A and B respectively, then OA =a, OB = b so that the co-ordinates of A and Bare (a, 0) and (0, 6). If M isthe mid-point of AB, then co-ordinacsofMare a+0 +b), (a b 2 2p \a2 But the co-ordinates of M are given to be (ry, yi) fo o oem and Bay, 4 = as2r, b=2y; Substituting these values in (i), the required line is xy ° 4 z in? 1 (a, 0) Example 12. A straight line moves so that the sum of the reciprocals of its intercepts on two perpendicular lines is constant. Show that it always passes through a fixed point. Sol. Let the two given perpendicular lines be taken as x-axis and y-axis, Let the intercepts of the moving tine be a and b so that its equation is Esker 0 By the given condition 44s consam= } (ay) 52 GOLDEN CO-ORDINATE GEOMETRY kik at which shows that the line (i) always passes through the fixed point (&, k). Hence the result, Example 13.04 and OB are two perpendicular lines. The straight line AB is dravm in such a manner that OA + OB = 8. Find the lacus of the middle point of AB. Sol. Take OA and OB, be two given perpendicular lines as y. co-ordinate axes. Let OA=0,0B=5 be the intercepts made by the line on the axcs. Co-ordinates of A (9, ) are (a, 0) Co-ordinates of B are (0, 6) Now OA+OB=8 b| a+b=8 @ Co-ordinates ofthe mid-point of AB ae given by a2t0 le ° ep “—o a=2r, b=2y. Substiteting these values ofa and b in (), we bave 2x+2ya8 or x+y=4 whichis the required locus of M. Article 2.9, The Perpendicular or Normal Form. To find the equation of a straight line given that “p' is the length of the perpendicular from the origin on the line and ‘a’ is the angle which this perpendicular makes with the positive direction of the x-axis. Or To find the equation ofa straight line in the form xeosa+y sina =p. Let AB be the line meeting the axes in A and B. Let ON be the perpendicular from © on AB, so that ON =pand ZXON= Let P(, y) be any point on AB, Draw PM 1 on the x-axis. Ye Then OM =xand MP =y wow Nvcosa = ON = OA cosa = p=(OM+MA) cosa = [OM + MP cot (90° - «)] cos & +: MAL - 2 EB = cot (90° ~ a) = (OM+MP tana) cosa -(e98 or pexcosa+ysina which is the required equation, Joos ‘THE STRAIGHT LINE 53 Remember. In this form of the equation (0 pis always positive, (i) the angle a may bave any value between 0° and 360° (iii) (co-eff. of x)" + (co-eff. of y)* = cos? a + sin? = 1. Example 1. Find the equation of the line for which p = 2, sina=£. 5 Sol. cotant=sirta 2 cosas eVindta a2 Vii aa VE ae8 w+ x008a + ysina =p becomes 3 4 fl »(=3)+»($)=2 ie 23redy=10 or 3x + dy = 10 and 3x=4y+10~0 |* Example 2. Find the equation of a straight line such that the length of the perpendicular from the origin upon it is 4 VZ units and the angle which this perpendicular makes with x-axis is tan! 1. Sol. Here paWE aster 1 a tna=1 w sina © Jy cosas Ze or sina = —Jp cosas Je 7 xcosa-+y sin a= p becomes Pete yen ve 1 1 oe appr ygyewt ie. x+ys8 or -xr-y=8 ie x+ysB or xtys—-8 =1 Example 3. Find the equations of two straight lines which are ata distance from the origin and pass through the point (0, 1). Sol. The equation of any line which is at a distance } from the originisxcosatysina=3. (i) “7 Itpasses through the point Q,1) +. sina =4 cosas2Vinsiva 21-4 = Putting these valves of sin. and os atin (i), we get s%Sretyet 27 *27"2 aVS.rsyel or and GOLDEN CO-ORDINATE GEOMETRY V3 .xty-1=0 , Vooeeyeiao| — Wtichare the required equations of the lines. Example 4. if pis the length of the perpendicular from the origin on the line whose intercepts on the ‘a’ and "b? ; ii1,2 aves are ‘a’ and b';show that “5+ 55. and B so that OA =a, OB =. or or Sol. Let AB be the line. Let it cut the co-ordinate axes in A Let OL. p be the perpendicular from 0 on AB. If ZAOL = a, then ZLOB = 90" -a From right angled triangle OLA, Pecoa of) From right angled triangle OLB, § = cos (90° -a) * e sina (i) ‘Squaring (i) and (ji) and adding, FB ecodarsitant tyra Dividing both sides by p*, eee Article 2.10. To reduce the equation Ax + By + C = 0 to the normal form x cos. +y sina = p. Sol. The given equation is Act By+C=0 @) Multiplying both sides by k, kAx + kBy + kC-= 0 (where k 40) KAx + KBy =-kC oO) If (éi) be the normal form of (i), then, (co-eff. of x)* + (co-eff. of y= 1 PAt+ PB = 1 or P(AT+ BY) =1 1 eR ks ‘Substituting the values of & in (i), we get A B c ” es +) Sy - - ol 2VAbeB? 2VAleB a VAT +B? #9 Equation (iii) now satisfies one property of the normal form. The second property that the right hand ‘side is positive will be satisfied ifthe ambiguity («) isteplaced by that sign which will make the R.H.S. positive. ‘Thus when C is +ve, the ambiguity must be replaced by minus so as to make the R.HLS. positive. A B c ‘The equation becomes — — VAl+Bt VAt+B?” VAl+B? And when Cis —ve, the ambiguity must be replaced by plus so as to make the R.H.S, positive, ‘THE STRAIGHT LINE 55 and A B -¢ ‘The equation becomes +B ys . V+ VATE VAT+B? Role. To reduce the general equation to the normal form: (Take the constant term to the R-H.S. and make it positive, if not so already, by changing the sign of every term. (i Divide each term by V(coelf. of x)’ + (coef, of). (i) Now coeff. of x= cosa co-eff. of y= sin a RHS. =p. (i) From the values of cos a and sin a, find the value of ct remembering the signs as shown in figure. Example 1. Reduce the following equations to the normal form indicating the values of p and a. @x+Vy+420 (x+y +570 (ii) 3 + 4y 4 1550. Sol. ()x + Vy +4=0 ‘Tearisposing the constant term to RHS. x+¥3y=-4 Changing the sign throughout to make the RELS. + ve —x-Viy=4 Dividing both sides by V(- 1+ (-V3¥ 52 weet wtp Byea fi Comparing () with -xeosa+ysina =p 4 ings SE cosa =—3 [-veh sin a= -"-ve] ps2 sin a and cos a are both — ve = cos 60° = cos (180° + 60°) = cos 240° ++ (becomes. cos 240° + y sin 240° = 2 which is the required normal from, Gi xey+ NT 20 ‘Transposing the constant term to RH.S. xtys-N2. Changing the sign throughout to make the R.H.S. + ve ~x-y= NZ. Dividing both sides by V(- 1)? + (- 1 ie. by VE we get yet 0 56 GOLDEN CO-ORDINATE GEOMETRY Comparing () with x cos.a.+ysina =p 1 . 1 cos c= — Be (— ve) sin =p (ve) ps5 J: cos-cand sing. are both ve. 2. ot lies in thind quadrant £06 om Jp = — 00s 45" w c08 (180" + 45°) w con 225°, “ a=225" (may be writen as —_xcos 225° +y sin 225° = 5 which is the required normal form of the given equation. id, Bt 4y+15=0 ‘Transposing the constant term to RHS. Bea dy=—15 Changing the sign throughout to make the R.HLS. + ve =3r=dy = 15: Dividing both sides by V(- 3)" + (- 4)" ie. by 5, we get -da-Y=3 onl) Comparing it with x cos a +y sina =p cos a= -3 (ve), sina = 3 (ve) p=3 + sin @ and cos a are both - ve ot lies inthe third quadrant. Equation (() is the required normal form of the given equation. Example 2, Reduce the equation V3x—y + 6 = O ofa straight line to the normal form. Find the values of panda, Sol. Please try yourself. [Ans. x cos 150° +y sin 150° = 3, p =3, a= 150°] Example 3. Reduce the equation x-V3y +2 = 0 to the form x cos.a.+ y sina. = p. Also find the values of panda. Sol. Please try yourself. [Aus. x cos 120° + ysin 120°'= 1,p = 1,a.= 120) STRAIGHT LINE IN POINT SLOPE FORM. Article 2.11. To find the equation of a straight line through the point (x,, y;) and having slope m. Sol. Let the equation of the required line be yame+e ~@ where m is given. Since the line passes through the point (x), y,), we have yam tc anfid) Subtracting (jd) from (@) [to eliminate e], the required equation is yoy = m(x =x). ‘THE STRAIGHT LINE 37 Remember. The general equation of a line through (x), y;) is D ‘when m can be determined from a given condition. Example 1. Fira the equation of the straight line through the point (2, 3) making an angle of 45° with the x-axis, Sol. HereQ=45* > m= tan@=tand5"=1 Also one point on the line is (2, 3) xs Required equation is. y—3 = 1(r~2)orx-y+1=0. Example 2. Prove that the equations of the straight lines passing through the point (I, — 1) and making angles of 30° and 150° respectively with the axis of x areV3(y + I} = + (x- 1). Sol. Please try yourself. Example 3. Find the equation of a straight line passing through (5, 7) and inclined at 45° to x-azis. If it passes through P whose y-co-ordinate is - 7, what is the x-co-ordinate of P ? Sol. As in Example 1, equation of the line is y=T=I@-5) or x-y42=0 -() Let the x-co-ordinate of P be h, then P is the point (/, —7). + (1) passes through. h-(-7)+2=0 of h=-9 ‘The x-co-ordinatc of P is 9, Example 4, Find the equation of the straight line which makes an angle of (90° + a) with x-axis and asses through the point (p cos a, p sin a). Sol. Here m =tan (90° +a) =— cota. One point on the line is (p cosa, p sin a). . Equation of line is y—p sin a =—cos.a (x— pcos a) m(p y—psina.=— S25 (— peos a) ysina—psin?o =—xcosa+pcos?a Xeosa+y sina =p (cos a+ sin’ a) xeosatysina =p. STRAIGHT LINE IN TWO-POINT FORM Article. 2.12, To find the equation of the line joining the points (x,, y,) and (3, ¥2) Sol. Let m be the slope of the line. Since the line passes through the point (x,, y,), its equation is Y=yr= max) (1) where x,y, are known and m is an unknown constant. If (1) passes through (1, ys), we have yo—yy= m(t)=4)) mao gees which gives m in terms of known quantities. Putting this value of m in(1), weave y—: ye 2a Fe a) which is the reqd. equation of the line. 58 g or or or GOLDEN CO-ORDINATE GEOMETRY Remember. The equation of the line joining two points (x), yi) and (2, y:) is: Example 1. Find the equation ofthe line through the points O{-5.0) cao (® (@ sec a, 0) and (0, pcosec a) ii) (at;?, 2at,) and (a1;?, 2a1,). Sel. () The equon ofthe ie hough 0) and 0) yroe 2 (x05) Here nae wsd m=O yom(r+5) or y=mrte. vse (i) The equation of the line through (p sec a, 0) and (0, p cosec «) is 10 ea CP mee) aL ye SEE Q—pecca) or yx 82 6 -pacca) ‘cosa ya SES (e—pace a) ory sina =—cosa x-ee ysin a =—x cos @+p orxcosa+ y sina =p. iii) The equation of the line = 6434 7k+3= 949 = 4k23 or kad ‘Hence the required ratio is 3:1 ie. 3:4. ‘THE STRAIGHT LINE 61 Exataple 12. In what ratio is the line joining (2, 3) and (~ 1, 4) divided by the linex + y #1 =07 Sol. Please try yourself. [Ans. 3 : 2 externally] Example 13. In what ratio does the straight line x — y - 2 = 0 cut the straight line joining the points (3,— 1) and (8, 9)? Sol. Please try yourself. [Ans. 2:3] Example 14, In what ratio is the line joining the points (2, 1) and (5, 2) divided by the line joining (3, 2) and (5, 0) ? Sol. The equation of the line joining the points CG, 2) and D(5, 0) is 0-2 y-2=g— 3-3) or y-2=-@-3) or x+y-5=0 @ If@ meets the line joining A(2, 1) and B(S, 2) in M a e') M divides AB in the ratio & ; 1, then co-ordinates of M arc * D{S, 0) Sk+2 2ked kel k+l |" AsMllies on (), 5k42 2k+l = 7 = arte 528 which gives k= 1. ‘Thus the ratio is 1:1 ie M bisects AB, Example 15, In whar ratio does the line joining (1, - 2) and (0, 1) divide the line joining (1, 2) and C59? Sol. Please try yourself. fans. 1: 1]. Example 16. fr what ratio isthe line joining A(1, 2) and B(4, 3) divided by the line joining C(2, 3) and Dé, 1). Sol, Please try yourself. fAns. 1; 1] 7. Jn what ratio does the straight line joining the points (7, 5) and (1, = 1) intersect the straight line joining (3, - 1) and (8, 9) ?Also find the co-ordinates of the point of intersection. Sol. Please try yourself. {Ans.2:356,3)] STRAIGHT LINE IN DISTANCE FORM. Article 2.13. To find the equation of a straight line passing through a given point (x, y,) and making an angle 0 with the xaxis in the form om yon cos8 sind where ris the distance of any potat (x, y) on the line from the polnt (x1, ¥:)+ Let Agi y:) be the given point and AB, the line through A making an angle O with the x-axis. Let P(x, y) be any point on Fine such that APsr. 62 GOLDEN CO-ORDINATE GEOMETRY Draw AL and PM perpendiculars on the x-axis and AK perpendicular on MP. Then AK =NM=OM—ON =r-x, Y. - KP =MP-MK=MP-NA=y-y; ZKAP = CXLP= 0 (conmsponding £) From the right angled triangle AKP, we have SS eosd =. r eo) ye _ Yow. . ssid. eo lid) From (i) and (ii), we have x=xtroas0; y=y.trsin® “Thus the co-ordinates ofthe general point on the line are (e1 #7 cos 0,94 +r sin 6). Remember : 1. (ty, ys) are the co-ordinates of the given paint. 2. O is the inclination of the Hine through (x,y) with positive direction of x-axis, 3. ris the distance of any point on the line from the given point (¢;, y,). For different values of r, we ‘get different points on the line. 4.ris + veifP is above A and negative if P is below A. 5. Co-ordinates of the general point on the line are (x, +r cos 0, yy + sin 0). ‘Example 1. A line is drawn through the point P(2, 3) inclined at 30° to x-axis. Find the co-ordinates of the two points on it at a distance 4 units from Pon either side, Sol. Co-ordinates of the given poi~* are (2, 3). Inclination with % a-axis = 30°, s 0=30° Let Q and R be the two points at a distance 4 units from'P’on either side of P. PQ=+4 and PR=-4 /. Co-ordinates of Q are (2+ 4cos 30344 sin30°) | Heresy =2 or ‘THE STRAIGHT LINE 6 Co-ordinates of Rare (2—4.c0s 30°, 3-4 sin 30°) Here x, =2 23 or (2-4. 3-4.5) nes - @-n5.1), @=30" Example 2. Find the co-ordinates of two poinas which are 2VZunits from the point (3, 4) and lie on the straight line passing through this point and inclined at an angle of 10 the x-axis. Sol. Co-ordinates of the given poit are (3, 4), Y Inclination with axis = AE = 135° = 138". Let Rand Q be the two points at a distance 2V2 from Poncither side of P. Thea” PRet2VE and PQ=-2v7 ‘Co-ordinates of R are G+ 2V2 cos 135°, 4 + 2VZ sin 135°) Here xj #3 or (@ + 2VE cos (180° -45°), wn24 0 4+ 2VZ sin (180° - 45°) r=wd or (= 2VF cos. 45°, 4+ 2VE sin 45°) 05 135° or (3-207-yo-4e207.3) or G-2,442) of (1,6) Co-ontinates of Q are (B-2E cos 135°, 4-25 sin 135°) Here x, =3 or @+2VE 00s 45", 4—2V7 sin 45") ved or (2+202-Jpe4-205-a) ne or +2,4-2) oF (5,2) Example 3. Find the distance of the point (2, 3) from the line 2x — 3y + 9 = 0 measured along a line making an angle of 45° with x-axis. Sol. Let the line through P(2, 3) inclined at an angle of 45° with x-axis mect 2x —3y +9 =0inQ. ‘We have to find PQ. Let PQ=r Co-ondinates of Q are [2 + rcos 45°, 3 + rsin 45°] or [+-3+75 * Qlieson2x-3y+9=0 . [2+ ]-3[3-99 4920 64 GOLDEN CO2ORDINATE GEOMETRY 2r 9 3r or 44 B-9-Fr9e0 or 4-Fen0 r or 4a or raavz which i the required distance. Example 4.A straight line is drawn through the point P(V3, 2) making an angle of 30° with x-axis. Determine the length ofthe line measured jrom ths point to where it meet the line V3x—¥) 48 =0, Sol. Let the line through P (VJ, 2) inclined af an angle of 30° with x-axis meet V3x- 4y +8= 0 in Q. Webave to find PQ. Y, Let PQ = r. The co-ordinates of Q are ° QT #7 cos 30°, 2 + rsin 30°) Boot or (3er$ 244) Qlies on V3x—4y +8=0 Bar F)-4(245) +520 or ve 34 $r-8-24 800 or — which is the required distance. . Example 5. Show that the distance of P(x, y;) from the;line ax + by + ¢ =-0 in the direction making ax, + by, +6 acosO+bainO",, « Sol. Let the Tne though P(t) and inclined at an angle with r-nxis meet the line ax + by +c = 0 inQ Let PQ =r. We bave to find r, Co-ords. of Qare (x +7-cas 8, y; +7 sin @) Qlies on ax+by+e=0 a(n +608 8) +b: +r sin®) +e=0 an angle 0 with xeaxis is = or (ary + by, +0) +1 (a 00884 bsinO)=0 or 7 (acon 0 + b sin 0) =~ (ax, + by, #0) Hence r= Zt 1+. wich is the requited distance: . , ine acos + sin 6" Example 6. A straight line passes through the point (2, V3) and makes an angle of 60° with x-axis. Find its equation, Also find the length of the intercept on this line between the point (2, V3) and line x + V3y = 12. Sol. One point on the line is (2, V3). Inclination of the line with x-axis = 60° slope = tan 60° = V3 aang poi slope form yyy zm(e—%), ‘ ‘the equation of the line is y-V3 = V3(x -2) or Viv-y+V3 -2V3 <0 of V3x-y-v3=0 CTHE(STRAIGHP.EINE) © = 1) 6s ‘Let thie line meet the line x + VJy = 12 in Q. ‘Let PQ =r. We havesto find r. . . ‘Co-ordinates of Q are Here -2y=2 Gsrinipnid staan ed or (2egatertd ) ae Qlics on'x + V3 y= 12 . (2g). (er se) -2 : or 245434012 of 2a? of raresd.. ‘Which ig the requized distance. Example 7.A line is drawn through the point P(3, — 2) and parallel tothe ine 2x +y + $= O which meets the line xy + 1 = 0 at Q. Find the distance PQ. Sol. If 6 be the angle which the line 2x-+ y + 5.5 0 makes ‘with the x-axis, then tan 6 = slope of the line 2 a2. Taking @, in the epcond.quadraat ap that sin 04s +.ve- end cos 0 is —ve, have L sin = 2, , cos = 7g + Since the line through P is parallel to ‘this ling, therefore, for the line through P, . sind Fy cone -gg oles POn ‘Co-ordinates of Qare ©! ~ . @ +rcos0,-2 +nsin 0) = ( 3= Te 2248) - vo B+ e520 Qed Grethe line xeyeted “38 2r car aan (2-ve)-(-2+%) +188 ‘ NS L Sametragetr or ers or fai PQ =2W5. ample. Show that he distance ofthe point - 1, 2) am the in 0:9 = 15 messared patel the line 6 — By + 1] = 0 is JO units. ot | Sol. Please try yourself. aun Example 9. Find the direction in which a straight line must be drawn through the point (1, 2) so that its point of intersection with the line x + y = 4 may be at a distance 16 from this point. Sol. Let the line through P(L,2) meet tetnc y= ia mech tht PQ“ elcid at on angle te a u is. ses GOLDEN COORDINATE GEOMETRY Goroninmes of Qare(1+ YB om 0,263 6 un Here x; =1,y1;=2 | ralvG,0—07 Qliesonx+y=4 1+} ¥B cos 042 +3 V0 sind a4 or 4¥6 (c050+ sinQ)=1 ot con + sin 8 = Fe Squaring, cos? 0 + sin? 0 +2.cos Osin 0 = 2 3 1+ sin 2003 sin 20 = 4 » sin 30° or sin 150° o 26 = 30° or 150° Hence @ = 15° of 75°. Note 5. If the equation of a line is y = mx +c and it makes an angle 8 with x-axis, then its alope wr » tan 8. tan A- tan B * a(A~B) 8 an Aten B tan (180"- 6) =-tan 8, ANGLE BETWEEN TWO LINES Article 2.14, Find the angle between the Hines y = mx +¢, and y= mx +¢, Sol. Let y = mx + €; be the equation of line AC which ‘makes an angle 0; with x-exis, 20 that m, = tan 6). Lety =m +¢zbe the equation of the line BC which makes an angle 0, with x-axis so that m, = tan O>, “cos? 8 + sin? = 1 23in 0 cos 0 = sin 20 3 The angie 0 between the lines is given by © = 0; 6) oF 180° —(8;-6,) + tan O= tan (0,~0,) or —tan @;—0;) ‘or tan [180° -(02~0,)] = tan (02-6) =—tan@—-8) oF tan(,~0,) = ten (8-02) tan 0; = tan By m ** Ty tan6, on 0,” * T+ m— mm, * tan O= T+ mm, “ ata Om tan | emmy Hence ‘THE STRAIGHT LINE 67 the! or is the required condition of parallelism of two tincs. ‘Note 1. in numerical examples, by the phrase “angle between the two lines”, we mean the ucute angle between. two lines, Note 2. [fone of the given lines be parallel to the y-axis, the above formula fails. Article 2.15. Find the condition of : @ parallelism (i) perpendicularity of the lines y= myx +6) and y = ox + ey, Sol. If 6 is the angle between the lines, then my ~ m T+ mma @ If the two tines are parallel, than 8 = 0 tan@ =tan0=0 wim 0 T+ mm my =m, = 0 or. mm =m, | Remember Remember: Two lines are parallel ifm, =n i.c. ifftheir slopes are equal. (i Ie the two tines are perpendicular, then @ = 90° tan 0 = tan 0° == mam foe Tem," roe L+ myn =0 then =O [Remember de. if fraction = © then its denominator = 0 is the required condition of the perpendicularity of two tines. the Remember : 1, Two lines are perpendicular if mm, =—1 ie. Ifthe product of their slopes = — 1 + my and m, are the negative reciprocals of each other. Hence, if two lines are at right angles, the slope of each is the negative reciprocal of the slope of other, ‘Thus, if the slope of a lines is 3. then the slope of any line 1 to it is—3. o _A ,. ~ Soxffofx [Nests Stop tine A #2, & -§ te | Article 2.16. Find the angle between the lines axeby+qe20 and areby+e2 Sol. Slope of the line ar oby +6, =O ism =— 5 Slope of the line eae + by +en=O lnm GOLDEN CO-ORDINATE GEOMETRY If Bis the angle between the two lines, then -4).(.2 tune | 2 5 Tem |" 7a) (a bb ait ba_by | | gabimarbs | | | erbr- arb 142192 | aya) +Bydy (0x02 + Byby bib i | aibz- adh Brees | aie biba |” Articte 2.17. Find the condition of () parallelism and (i) perpendicularity of the lines axtby +c 20 and axehy+ose Sol 10 isthe angle between the line, then tan siba~ aby (Ire two tines are parale, = 0° tan Oe tan0=0 = sto => aybp—ab\n0 = ab aad) ep Which is the required condition of parallelism. “+ the lines are parallel ifthe ratio of co-ffcients of x = ratio of co-efficients ofy. (DAEs wont a a i anh 0 = 907 tan @ = tat 90° = co = Bane ae => 0409+ bby x0 which isthe required condition of pexpendiculatty, the lines are at right angles if, Product of co-efficients of x + Product of co-efficients of y = 0. Example 1. Show thet she angle between the lines whose intercepts on the axes are eb and a’, is ab! -a'b ta Sol. The equation ofthe line whose intercepts on the axes are a and b is 2,2. yy, atten! © | rezcept on rants * inersopt on yanis "> 1 Its slope mie aT . m=-8. ry ‘THE STRAIGHT LINE o ‘The equation of the line whose intercepts on the axes are a! and b' is aepet i) Its slope were “CHE tang 222d Hence 0 = aa + BS Example 2 Find the angle between the lines xecosatysinasp, and acosf+ysinb=p’.[a>B] Sol. Slope of the line x cos a +y sin a =p is mae, Slope of the line.x oos B +y sin B =p’ is m= SEE ‘If Gis the angle between the lines, then cos, cos ~Gaa‘ sin = ey _ Sinacos B cos.a.sinB sin (o- B. tan (0-8) “Tema Ty =} sin asin B + c08 a.cos fi" cos (a i) chads ‘tan Hence = O=a-f. Exaraple 3. Find the angle berween the straight lines Zyhel and ZY 6 brant Sol. Slope of the first line ism, = y+ The lines are perpendicular to each other. Hence angle between the lines = 90°, Example 4. Find the angle between the fines (2-y43=0 and xty-250 (i) x42y+9=0 and Sx+y-7=0, 70 GOLDEN CO-ORDINATE GEOMETRY Sol. (2) Slope of the first line is Slope of the second line is me-te-1 4 = HOt Menge Berroa He, em nO = Tm Ted TE 6s the acute angle between the lines then tan p= tan (180°-0)=—-tan@=3 => ¢=tan'3 (Gi) Please try yourself. - (Ans. 45°] ‘Example 5.A(1, 2), B(- 3, 2) and C(3, - 2) are the vertices of a triangle. Find tan A, tan B, tan C. Sol. Slope of AB is. matteo Slope of BCis m= --2 Slope of CA is martin 2 ‘Angle A is the angle between AB and CA. ‘ tan Ae 71="3 042.4 => Asun'Qy Tem; 1+0 Angic B is the angic between AB and BC. my — Ms 32 tant (2 B= Tm 14073 = Beus (3) Angle C is the angle between BC and CA. 2 moms ast? Tem 14-2) Article 2.18. To find the equation of a straight line parallel to Ax + By +C =0. Sol. Let the equation of any line parallel to Ar+By+C=0 be Aw+By+C,=0 ‘Then ALB Lean * Ay + kA, By =iB s0 that (ii) becomes, kAr+ kBy +, =0, Dividing it by &, Are ByeDa0 * tan 4 2324 stant (4 arf > Cnt (#)- or Ar+Byt+th=0 [wens or $] Hence any line parallel to Ax + By + C = Ois Ax + By +2. =0. ‘THE STRAIGHT LINE n Role. 1. Keep the terms containing x and y unaltered. 2 Change the constant. 3. The constant 2. is determined from an additional condition given in the problem. ‘Thus the equation of any Hine parallel to 2x —3y +5 =O is 2x —3y + =0. Article 2.19. To find the equation of a straight line perpendicular to Ax + By + C= 0. Sol. Let the equation of any line perpendicular to Ar+By+C=0 onli) be Ag+ By + Cy 20 fii) ‘Thea. AA,+BB,#0, or AA, =—BB; se Beep “ Ay =kB, By=—kA so that (i) becomes, ABx-kAY+ C, =0 Dividing it by &, Br-Ay+ ao or Br-Ay+h=0 [wstine ror] Hence any line perpendicular to Ax + By + C= Ois Bx-Ay +h= 0. Role, 1. Interchange the co-effs. of x and y and change the sign of one of them. 2. Change the constant. 3. The value of A. can be determined from an additional condition given in the problem. ‘Thus, the equation of any line perpendicular to Zc -3y +50 is 3x+2y+2=0. Exaraple L. Find the equation of the straight line passing through the point (1, 1).and parallel to the Hine 4x + 4y + 7= 0, Sol. The equation of any line parallel to dr ¢4y+7=Ois dr4dy+he0 If it passes through the point (1, 1) ;then444+h=0 or 4=-8 Equation of the required line is4x+4y-8=0 or x+y-2=0. Example 2. Find the equation of the straight line passing through the point (4, —5) and perpendicular to the line 3x + 4y 45 = 0. Sol. The equation of any linc perpendicular to 3x4 4y+5=Ois 4x~3y+h=0 IF it passes through the point (4, 5), then 4(4)—3(-5)+A=0 or A=—31 Equation of required line is 4x ~3y-31= 0, Example 3. Find the equation of the straight line through the point (~ i, 2) and (@ parallel tothe line 3x~y = 6 (Gd perpendicular to the line x - 2y = 8. Sol. Please try yourself. [Aas () 3x-y+5=0(i) & +y=0) ‘Example 4, Find the equations of the lines passing through (4, 5) and (© parallel to and (id perpendicular to 2x + 3y = 5. Sel. Please try yourself. [Ans. (2x + 3y = 23 Gi) x—2y = 2.) Example 5. Find the equation of the straight line perpendicular to the join ofthe points (2, 3) and (4, = 2) end passing through the point (2, 5). Sol, Slope of the join of A (2, 3) and B(4,- 2) is e223 Ss 24 "22°72" ed GOLDEN CO-ORDINATE GEOMETRY +. slope of any line AB is (ve reciprocal of the slope of. 2 sina AB)=3. re [Pes If it passes through the point P(2, 5), then its equation [Using point slope formy=y1=m(x—x)}is = . ; * - y-S=He-2) nt eee aie tet So rat Lt dans ke. Sy-25=2x-4 es . or 2x-Sy +2150. oA 8. 2.3) (4-2) Example 6. Find the equation of the straight line which bisects and is perpendicular tothe line joining 2,3) and (4, 5). or er Se rat eee co ea me le eee ee ee eee deg aes difference of ordinates -2 5. Slope of AB = Gitrerence of abscissac "24" ' 2. Slope of any line AB =~ 1 © [+weseciprocal). 4-5, ‘Co-ordinates of the mid-point M of AB are: 244 24s . A ( om fe G4 (2.3), ” po ee y-4=-1@-3) Using point-slope form . af M 8 G4) (4,5) or x#y-750 y=y15 m(x=2)). : . te nal Example 7, ind the equation ofthe Fight bisector ofthe in of the nts (;3)and (3, 45). Sol. Please try yourself. os Ams x4 dy -6 = 0] Example 8. The vertices of a triangle are A(10, 4); B(—4, sya ce 2, ~1).\Find the equation of the altitude through A. Also. find the foot of this altitude. ...\..:.<1- »: Sone a aber Sol. Altitude AL is the line through A 1 BC. stope orc = 22. 214 5° 8 AAO, 4) 4-(2) bik + is * ent [-ve reciprocal] Equation of the altitude AL is A is y-4=i(@-10) | ALisalline through or “Sy-20=x-10 ‘A(IO, 4) with slope 3 or = x-Sy41020 > +) Use poitt-slope form * Te find L, the foot of this altitude. Equation of BCis or y-9n-S(x+4) or Se+ye =O Also equation of ALisx=Sy#10=0 THE STRAIGHT LINE 3 \ csciueL being the point of interseetion of BC and AL is obtained by solving (1) and (2) simultaneously for wand y z7ly 1 Thos 10+55 "11-50" -25-1 <. Listhe point (3, 3). - Example 9. Find the equation of the straight line perpendicular toy = 3x-2 and making an intercept 4on the y-axis, Sol. The equation of the given linc is y=3r-2 of 3e-y-2=0 ~) Equation of any line Lto(i)is x+3y+heO wali) ‘obtained by putting x = 0 = xe-3yysi Its intercept on y-axis But it is given to be 4, - . hes or Re-12 Putting this value of & in (fi), we get ae x43)=12=0 which is the required equation the line. Example 10. Find the equation of the straight line parallel to 2x + 3y + 11 = 0 and such that the algebraic sum of the intercepts on ihe axes is 15, Sol. Equation of any line parallel to 2x + 3y + 11 =O is 2c4 3y+h=0 ) Intercept of () onsaxis=—% by puting y=0 Intercept of (jon yaxis=—* | by punting r=0 ‘Suu of these intercepts = 15 [siven] Putting this value of 2 in (7), we get 2x + 3y — 18 0 which is the required equation of the line. Example 11. Find the equation of the straight line perpendicular to 4x + 7y + 9 = Qand such that the triangle formed with the axes has area 3.5, Sol. Equation of any line 1 to dr + 7y +9 = Vis 7x-4y+A=0 a0) It meets a-axis where y = 0 ¥ Putting y =O in (), we get Tx = —h xs- 7 n= 3 5 (i magni) Also (i) meets y-axis where x = 0: Putting x = 0 in (i), we get ~4y thd te. yee . oe oo OB 4 1s GOLDEN CO-ORDINATE GEOMETRY Now ares of AOAB = 3.5 (piven) 1 Afeyiz 2. 2 $0A.0B=35 or [3 =z or N528x7 or 175196 ‘ hee Putting this value of & in (), we get 7x—4y = 14=0 ‘Which are the required equations of the lines. Example 12. Find the equation ofa straight line drawn at right angles tothe lines = +% x I through the point where it meets the axis of x. Sol. The line Beat - —o ™ micets the x-axis in A whose co-crdinates are (@, 0). Equation of any line 4. to () is Z-E+a=0 -ié) [rit passes throvgh the point A(a, 0), then aio 1 a prathee te he-e 2: (ii) Becomes got Fe0 ie ax—bysat which is the required equation of the line. Example 13. The perpendicular from the origin to a straight line meets is at the point (b, k). Show that the equation of the straight line is ha + ky = +P. Sol. Let AB be the line whose equation is required. Let OL be 1. AB through the origin O(0, 0), & differeace of ontinates k Slope of OL =F ik “of abeci .. Slope of AB which is 1 to OL. h = [-ve reciprocal] ABis a line through L(t, A) with slope — Equation of ABis yoke tea | Point-stope form or iy-Pe-letl? or bets +P. Example 14. Prove that the equation to the siraight line which passes through the point (a cos’ 0, a sin? 8) and is perpendicular to x sec @ + y cosec 8 = a is.x cos 8 ~y sin ® = acos 20. Sal. Slope of the line x see 0 + y cosee O =a sec 8 sin @ i “ye SED. Slope of any line 1 to it == ‘THE STRAIGHT LINE 6 + Equation ofthe line through (e cos, a sin? 0) and having slope S22 is yrasit 0 = 22 (a cos?6) or ysin @—asin*@ = rcos 0—acos'@ or 008 0—yssin 8 = @ (cos"0 —sin* 0) = a(cos? 6 + sin’ 0) (cos? 0 sin? 0) = a(Xo0s 20) or xcos@—ysin 8 = acos 20. Example 15. Find the equations ofthe two straight lines passing throtigh the point (1, ~ 1) and inclined at an angle of 45° to the line 2x ~ Sy + 7= 0. Sol, Slope of the line 2x - Sy +7=0 wf) P(1,-1) i 2 coeff. ofz is 3-Sote| Let m be the slope of any line through P(1, 1), then its equation is 2) [S y-()=m(x-1) [Point-slopeform 4 @ 2 -Sye7-0 RB or yelem(r1) li) Since (i) and (ii) are inclined at an angle of 45" | given 2 tan 45° = 2 ——- i+m@) tea 2 aad * 54am (i) Taking the + ve sign, we get S#2m=Sm—2 or 3m=7 - m=} Putting this value of m in (i), ytte2@-1 ie. Te-3y-1020 (i) Taking the —ve sign, weget 5+2m=-Sm+2 or Tm=-3 me 3 Patting this value of m in (it), yrta Senn je 3r+Ty+420 ” Hence equations of required lines are 7x-3y-10=0 and 3x+7y+4=0, Example 16. Find the equations of two straight lines through the point (4, 5) which make an angle of 45° with the line 2x-y +7 = 0. Sol. The given line is 2x -y +7 =0 (i) Pia, 5) its slope = 2, . Let m be the slope of any line through the point P(4, 5). Then its equation is y-S=mfe-4) ii) |Points-slope form LQ las Since (i) and (ii) are inclined at an angle of 45° |given A Q = 2&-y+7-0 OR B one maz my ~ ma olan 45° = 14m) . (nO = =a = i2 i+2m or 1 16 GOLDEN CO-ORDINATE GROMBTRY (i) Taking the + ve sign, we get, = 14+ 2m=m-2 or nis -3 Putting this valuc of m in (ii), Bend) ie, e+y—17=0. (ii) Taking the — ve sign, we get 142m=—m+2 or 3m=1 or me} Putting this value of m in (i), y-S=l@-4) ke x—3ytte0 Hence equations of required lines are 3x +y—17=0 and x~3y+ 11=0. Example 17. The base of an equilateral triangle is the line x + y = 2 and the veriex is the point (2, - 1). Find the equations of the other sides. Sol. The base BC of equilateral AABCisx+y=2 (1) 1 A2,-1) lis slope ==} ==1 Letim be the slope of any line through the vertex A(2, ~1). Then its equation is y-Cl=m@-2) [Point-stope form or y+ te m(e—2) 2) ‘Since AB and AC are lines through A inclined at 60° to BC. Angle between (1) and (2) is 60° Ast eur\ - tan 60° Seren met 8 xeyed c (0 Tenleg + nan oe gad — memes wi =1=(v3+i)m or m VB-1, v3-1_ 341-27 1) og a meee VS-1—> ~ Putting this value of m in(2),wehave = y+ 1 = (2-V3)(x-2) (i) Taking —vesign, weget— VT-VEm=—m—1 or S412 (05-1) m of m= ett VE41 V5+1_ 361465 or mR Set =2+V0 Putting this value of m in(2), we have = y + 1 = (24+ V3)\(x-2) Hence the equations of the other two sides are y + 1 = (2 # V3\(x-2). Example 18. Find the equation of the two lines passing through the point (1, — 1) and inclined at an angle of 60° toY3x-y + 7=0. Sol. Please try yourself. [Ans. V5xey=V3-1y41=0) Example 19. Find the equations of the two straight lines passing through the point (2, -2) and inelined at an angle of 45* to 2x-y + 1 = 0. Sol. Please try yourself. [Ans. 3c +y = 4,2—3y = 8] Example 20. Find the equations of the lines passing through the point (2, 3) and making an angle of 60° with the line x=y +5 = 0. Sol. Please try yourself. [Ans. y-3=-(@ sV3\(r-2)} Example 21, Find the equations of she straight lines passing through the point (2, 3) and making an angle of 45° with the straight line 3x + y = 5. Sol. Please try yourself, [Ans.x#2y = 8, 2r=y= 1) Example 22. Find the equations of the straight lines passing through (3, - 2) and inelined at 60° to the straight line V3x+ y= 1. Sol, Please try yourself, fans. y+2=0,VEx-y=2+3V3] ‘THE STRAIGHT LINE 7 Example 23.4 diagonal of a square lies along the line &x ~ 15y = 0 and one vertex of the square is (1, 2). Find the equations of the sides of the square passing through this vertex. Sol. Let ABCD be the square. on. c Let &r- 15y=0 sufi) C7 be the equation of the diagonal AC and let D(1, 2) be the given vertex. ‘We have to find the equations of the sides AD and CD. Since the diagonal of a square is inclined at 45° to the sides. + the required lines are the Lines through D(1, 2) inclined at 45° to Br ~ 15y =0. 8 Slope of AC= 75. [> Let m be the slope of any one of the required lines through D(1,2),. 8 then its equation is y—2 = m(r — 1). ii) Angle between (i) and (ji) being 45°, _8 oe 15 iim-8 tan 45° <7 gy vl 8 }* 715+ 80 itm(= 15 (i Taking + ve sign, we get 15+ 8m=15m-8 or Tm=23 nad, Putting this value of m in (i), y-22 Be 1) of 23x—Ty-9=0. i) Taking —ve sign, we get 15 + 8m=—15m+8 of 23m= Putting the value of m in (ii), pod (r= Norte+ 239-53 =0 Hence the required lines are 23x—7y—-9=0 and 7x +By~53=0. Example 24, Two opposite vertices of a square areA(],~2)and C{- 5, 6). Find the other two vertices. Sol. Let B and D be the other vertices whose co-ordinates we have to determine. D - Let M be the point of intersection of the diagonals. Rt 8-8) As Mis the mid-pointof AC [- diagonals of a square bisect +6 2 ie. cach otber at right angles}, its co-ordinates are ( 22. Also, diagonals of a square are equal. BD = AC=V(145)*+(-2-6) =Vi6e64 = 10 +) M isthe mid-point of BD. AU.—2) 8 10 MD=MB =" =5 Slope of AC 78 GOLDEN CO-ORDINATE GEOMETRY Slope of BD which is 1 to AC = 3 [-ve reciprocal]. If 0 is the inclination of BD with x-axis, then tan a 3 4 so that sind=3 and cos = 5 3 Now consider the line BD, M@2, 2) isa known point on this line. LA Inclination of BD with x-axis is 0 where tan 0 = 4 ig LGD yn? Equation of BD is "ooh Le ar = xe-2srcos0, y=2+rsind B and D are at adistance of $ units on cither side of M. Putting r = = 5, the co-ords. of the remaining two vertices are (2+ 5 cos 0, 2+ 5 sin 6) and (-2-S.cos 0, 2-5 sin 0) ies (-245-$.245.3]ani(-2-5.5.2-5.5 ie, (2,S)and INTERSECTION OF TWO STRAIGHT LINES Article 2.20. To find the point of intersection of the lines axthyre.20 and ax +hy +e,=0, |. Let (ct, ) be the point of intersection of axtby te =0 and artbyte,=0° ‘Then as the point of intersection lies on both the lines ‘ aa +b,B +0) =0 agar botc2=0 Solving by cross-multiplication, So Bye-bye ey aya; dyer = Baer | | C12 = Cty 1; ~azb;"" ~ ayby— ab bye bre, eye ene ‘ayby— dab " ayby ~ pb, Remember, The point of intersection is obtained by solving the two given equations simultancously. The values of x and y so obtained ate the abscissa and ordinate of the required point of intersection. ‘The point of intersection is Example I. Find the point of intersection of the lines ar + by = 1 und be + ay = 1. Sol, The equations of the two lines are ax+ by bx + ay— =0, =0 ‘THE STRAIGHT LINE: 0 Solving by cross-multiplication, x 1 Tbea -bta gob 2 yy ~__ 1 _ a=b a-b (a+b\a-b) 1 1 + depotetinenecini( 45). or or xays ath Example 2. Show that the line 3x + 3y + k = O passes thraugh the point of intersection of 3x + 4y +6 =O and 6x + Sy-9 =0ifk=-T. Sol. The point of intersection of the lines 3x4 4y+6=0, and Gr + Sy-9=0 xy 4 = 36-30 * 36427" 15-24 is given by or or xa 22 xed, . - (22 Point of intersection of (i) and (fi) is ( 2. ) If the line 3x + 3y + & = 0 passes through it, then {F)scnse-0 or 22-21+k=0 of k=-1, Article 2.21. Line through the point of intersection of two given lines. Sol. Let ar tby +, 20 () and ant by +e)=0 watt) be the two given lines. Let (i) and (ji) intersect in (c, ), then (ct, B) lies on both (i) and (ii). ayant biB+ c= ® alii) ay + bo +09 = (i) Consider the equation (aye + by + ¢)) + Mex thy +e)}=0 (0) It is a first degree equation inx and y. it represents a straight line. Multiply (iv) by 4 and add to (iii), (ae + 6,8 +c,) +A (ax +b +e) =0 — whatever Amay be. This shows that (a, B) lies on (1). Hence the equation (v) represents the equation of any line through the intersection of (i) and (ii) which is a fixed point. Asticle 2.22. Show that, whatever numerical value be given to the constant 2, the equation Kayx + by +e)) + Mags + bay +e) = 0 represents a straight line through a fixed point. Determine } so that the above straight line may be perpendicular to the straight line age + bay + ¢y= 0. 80 GOLDEN CO-ORDINATE GEOMETRY Sol. For the first pany, see the above article. Slope of the line ay + by +c=0 is is Fe - a + ary Slope of (v) hee If (vi) is perpendicular to (v), then product of their slopes = — 1 43) ( i+ daz) = > (-#)( by bby 1 = aay + hay) = - by(h, + Ady) = Mazts + babs) = — (ayy + Bibs) _ ayaa t bby Hence 1 hie Exercise. Show that the equation (a,x + byy + cj) + Mazx + bry + €3) = 0 represents a straight line ; ; _ bier baer ey + ext passing througlea feed point. Find ats he co-ordinates ofthe fixed point | Ams. so oe WU, =0, . . . wo . . Remember. u co | #8 the equations of two lines then any Line through their point of intersection is U) #2U3 = 0. dis called parameter. Any particularline "in fact an infinite number of straight lines can be drawn through the point cf intersection] is obtained if one more geometric condition is given to determine A. Example 1. Find the equation of the line through the poine (3, 4) and the point of interseetion of the Hines Sx-y=9 and x + 6y 28. Sol. The given lines are Sy-y-9=0 m=0 and x4 6y-8=0 uy=0 “The equation of any line through their point of intersection is (Gr-y-9)+ het 6-8) =0 eff) [uy t hu = 0 I it passes through the point (3, 4), then (19 -4-9) 4234 24-8)=0 or 24192=0 or rez Putting this value of & in (i), we have 2 (Sx-y-9)—75 (4-8) =0 ot 95x - 19y—171 —2e— 12y 4 16 =0 Br -3ly-155=0 or 3x-y-5S=0 he required line, Example 2. Find the equation of the straight line which joins to point (1, 1) to the common point of 2x-3y- HM = Oand x~3y-22 = 0. Please ary yourself. [Ans.x-y=0] Example 3. Find the equation of the straight line through the point of intersection of the tines 2x + 3y +5 = Oand 3x +-4y—18 = 0 and paraltel to the line 5x + 2y + 9 = 0. ‘THE STRAIGHT LINE at Sol. The given lines are 2x + 3y + m=O and 3r44y-18=0 y= 0 ‘The equation of any line through their point of intersection is (2e+ 3y +5) + 0Gr + 4y—18)=0 voi) [eq +a =O corm. ofx | 243k coeif.ofy 344k If @) is parallel to the line Sx + 2y +9=0 Its slope =~ whose slope = 3 then slopes of (i) and (ii) are equal 2+3k_ § 1a ' - SE Gn rr Ord + he 1S + On condition of parallelism m, = mz = --ll or Mae 1 or Remy Putting this value of 2 in (i), we get (e+ sy+5)-th r+ 4y - 18) 30 or 28x 4 42y 470 -3ar—ddys 19820 or —5x—2y+268=0 or Sx+2y=268 whichis the required line. Example 4. Find the equation of the straight line through the point of intersection of the lines ax + by 4+¢ =Oand a'r +h'y +c’ =O drawn parallel to.x-axis. Equation of any line through the paint of intersection of the Tines artbytc=0 and a'rtblyte'=0 is art byte+da'rs by +e)=0 or (2+ dale + (b+ Diy 4 (C+ 2c) =0 (i) IF itis parallel (o x-axis, then it must be free from the term containing.x. coeff ofreatha'=0 iehk= Putting this value of} in (i), the required equation is (o-5 wyelens }r0 or (a’b=ab’y + @'e =a’ Example 5. Obtain the equation of a straight line passing through the intersection of 2x - 3y + 4= 0 and 3x + 4y = S and drawn parallel to y-axis. Sol. Please try yourself. [Ans, 17¢ + 1 =0] Example 6. Find te equation to the line passing through the intersection of x + 2y + 3= 0 and 3x + dy + 7= Gand perpendicular to the tine y =x = 9, Sol. The given lines are x 42y43-=0 m=0 and 3x +4y+7=0 n=O The cquation of any line through their point of intersection is (+2p43) 4 AGr+ 4y47)=0 2 uy +), =0 Its slope =— See of 1+ 3h coll, ofy 244% 82 GOLDEN CO-ORDINATE GEOMETRY If @ is perpendicular tothe line y—x=9 (ii) whose slope = then the product of the slopes of (i) and (ii) is equal to 1 143d ~T+4h Putting % = —1 in (i), we get (e+ 2y+3)-Grtdyt a0 or It2y+4=0 or x+y+2=0 which is the required line. Example 7. Find the equations of the straight line through the intersection of 2x + 3y +45 Oand 3x + 4y-5 = 0 and perpendicular to 6x -7y +8 = 0. Sol. Please try yourself. [Ans. Tx + 6 = 85] Example 8, Find the equations of the straight lines through the point of intersection ofthe two straight xle-1 or 143k2#2+44)0rk=-1 lines x + 2y-3 = 0, 3¢ + 4y +7 = Oand (@ parallel io y- 2x +8 = 0 (id perpendicular to y + x +4 = 0. Sol. Please try yourself. [Ans. ()) 2r—y 434 = 0 (i) 3r—y 4.47 = 0} Example 9. Find the equation of the line passing through the point of intersection of x -2y + 5 = 0 and 3x + 2y +7 = Oand perpendicular to xy = 0. Sol. Please try yourself. [Ans.x +y +2 =0] Example 10. Find the equations of the lines passing through the point of intersection of 2x + y= 3 = 0 and 3x = 4y + 7 = 0 and (8) parallel to the line x -5 = 0 (i) perpendicular to the line x + y = 7. Sol. Please try yourself. [Ans. (i) 11x —5 = O (ii) 1ix— Ly + 18 =O} [Hint. Any line parallel tox - 5 =0 isx=& ie. term containing y is absent.] Example 11. Find she equation of a straight line that passes through the intersection of x—2y = aand x+ 3y = 2aand is parallel to the line 3x + 4y = 0. Sol. Please try yourself. (Ans. 3r + 4y = Sq] Exaruple 12. Find the equation of the straight line through the point of intersection of x ~2y +3 = 0, 2e—y +3 = Oandparailel to the line 3x—4y + k= 0. Sol, Please try yourself. [Ans. 3r—4y +7 =0] Example 13. Find the equation of the tine through the intersection of y +x = 9 and 2x -3y +7 = 0 and perpendicular to the line 2y = 3x5 = 0. Sol. Please try yourself. [Ans. 2e + 3y- 23 = 0] Example 14. Show that the equation of the line joining the origin to the point of intersection of 2 yYepand=+Xelisre Te petands she lisxey. Si i Eygtiie 242 ye Sol. The given linesare Z+f—1=0 and 542-150 ‘The equation of any line through their point of intersection is yy Evia). (i+ ror(srs 1) 0 ~@O If it passes through the origin @, 0), then -1~-=0 or A==1 ‘THE STRAIGHT LINE. 83 « o(S-f)-o(be$]e0 oe cope te rey which is the required line, Example 15. Find the equat*-*s of the straight tines through the intersection of 3x —4y + I = 0 and Sx + y= 1 = O which cuts off equal intercepts from the axes. Sol. The given tines are 3x—4y + 1=0 and Sx+y-1=0 Equation of any line through their point of intersection is Gray +1) 4245e¢y—1) 20 @ or G+ Set QA t 1 -h=d Its intercept on.x-axis is obtained by putting y = 0 ” G+sen-1 or re teh 345y 4 hed + itercepton x-axis = 5 Intercept on y-axis is obtained by putting x = 0 in (i) ” Q-dy=A-1 or eho! Intercept on ynaxis = 54 If these intercepts are equal, then or A-4=2(3+5) 74 a Putting =—2 in (), we have @r—4y+1)-LGery—1) 20 or 23(r+y)=1 which give 4 = Puning = 4 in (), we have Grave 4 Lx ty-t=0 or 23@e-y)+5=0 Hence the equations of the reqd. lines are 23(x + y) = 11 and 23(x-y) +5=0. Example 16. Find the equations of the straight lines which pass through the point of intersection of the lines 3x —4y + 6 = O and 4x—y—5 = 0 and cut off equal intercepts from the axes. Sol. Please try yourself. fAns.x—y+1=0,x4+y-5=0] Example 17. Find the equation of the line joining the point of intersection of 3x + 2y + 1 = Oandx + y =3 = O10 the point of intersection of 3x + 2y -1 = Oandx +y-5=0. Sol. The point of intersection of 3x+2y+1=0 =) and x+y-320 lf) is given by — eT Point of intersection of (i) and (ii) is (~7, 10) Equation of any line through the intersection of 3x42y-1=0 and x+y-5=0 is r+ 2y-I)+ Mert y—5p=0 -Aiti) 1y=10 84 GOLDEN CO-ORDINATE GEOMETRY {fit passes through the intersection of (f) and (ji) ée. through (~7, 10) then 21 +20-1)44C7+10-5)=0 of -2-220 of A=-1 i), we have Gr42y-1)-(e+y=5)=0 ie 2e+y+4= 0 which isthe required line. Example 18. Find she equation of the straight ine whieh passes through the intersection of the straight lines 3x + 2y +4 = 0 and.x—y = 2 and forms the triangle with the axes whose area is 8 Sol. The equation of any line through the point of intersection of the lines Be+2y+4e0 and x-y=2=0 Putting = is Gx 4 2y 44) +Xe-y-2)=0 or (3 +d) + (2-y + 4-DW)=0 ; js w 2A 4 . Its imercept on x-axis = 5 [by putting y = 0- Its intercept on y-axis | by putting x = 0 Area of the triangle formed by (1) with the axes is 8 — = 2. 2 =h= > TS Tes > APH 10A4 16 = 166-2 ”) = 20x7=80 Aas? When 2 = 2, (I) passes through the origin (each intercept = 04) Rejecting it and putting & = —2 in (1), the reqd. Tine is Gr +2y44)-2e-y-2)=0 or x4 4y48=0. [Note. Orthocentre isthe point of concurrence to the altitudes of a triangle.) Example 19. Find the co-ordinates of the orthacentre of the triangle whase vertices are (1, 0), (2, ~ 4) and (~ 5, ~ 2). Sol. Let A(1, 0); BQ, —4) ; C(-5, ~ 2) be the vertices of AABC. Draw AL 1 BC and BM LCA Onthocentre H is the point of intersection of altitudes AL, and BM. M To find the equation of AL. Slope of ne = SA AU, 0) 7 8 c 1 (2-4) (5-2) Slope of AL which is L BC = > (- ve reciprocal) Equation of ALis = -y-0 =i @-1) Point-stope form or Tx=2y=7=0. O To find the equation of BM. -2-0 Slope of CA==2=9 = 4, slope of BM which is L CA =—3 ‘THE STRAIGHT LINE 85 Equation of BM is y~(~4) =-3(r-2) or 3x+y- From (i) and (ii), by croslipticaon =0 Gi) a a a My .st Brae Tes 8 BOB Ho? onmcnen( #2), Example 20. Find the orthocentre of the triangle whose vertices are (0, 1), (I, ~ 2), 2, —3)- Sol. Please try yourself. [Ans. (~7,-6)] Example 21.4 triangle is formed by the lines y # x - 6 = 0, 3y =x + 2 = 0, 3y = Sx + 2. Find the co-ordinates of its orthocentre. Also determine the length of the altitude on the side whose equation is y + x-6 = 0. Sol. The equations af the sides BC, CA, AB of AABC are xty-6=0 x-3y-2=0 Sx—3y+2=0 A is the point of intersection of CA and AB. ime (i aa Solving (ii) and (iii), ere s* or xe-Lye-L Co-ordinates of A are (-1,~1) Cocif.ofx Slope of BC = -1 ~ Gociofy < Slope of AL which is 1 BC = 1 Equation of ALis y+1=1(e+1) — | Point-slope form or x-y=0 =v) B is the point of intersection of AB and BC Solving (/) and (iii), ye or r=2,ye4 Co-ordinates of B are (2, 4) Slope of CA = 4 <. Slope of BM which is 1 CA = -3 Equation of BM is y-4=-3 (x2) [Point-slope form or ax+y-10=0 on) Solving (iv) and (v), we have x=y= é ontocae is, $) Length of altitude AL = Length of 1 from AC 1, — 1) on BC -1-1-6 ary thy te + Var +? = —4VZ = 4vJ (in magnitude). 86 GOLDEN CO-ORDINATE GEOMETRY Example 22. Prove that the point (— I, 4) is the orthocentre of the triangle formed by the lines whose equations are x—y + I = 0, x~ 2y +4 = Oand 9r—3y +1 = 0. Sol. Please try yourself. Example 23. Find the foot of perpendicular from the point (2, 3) on the line x = 3y +4. Sol. Let x2¥y+4 ie x-3y-4=0 i) be the equation of the line AB and C(2, 3) be the given point. Draw CD AB, so that D is the foot of perpendicular from C on AB, Equation of any line L ABis 3r+y+#A=0 ii) If it passes through C(2, 3), then 64342%=0 or 2 Putting 4 =-9 in Gi), oety- ‘which is the equation of CD. D is the point of intersection of AB and CD. Solving (@) and (ii), we get x 27+ (2, 3) 1 1249" 149 i 313 Co-ordinates of D (ipa): Example 24. Find the equation of the perpendicular from the point (I, ~ 2) on the line 3y = 4x~5. Also find the co-ordinates of the foot of the perpendicular. Sol. Please try yourself. Ans.3e+4y +5=0;% Example 25. The line 2x-3y-4 = Qis the perpendicular bisector of the line AB and the co-ordinates of A are (- 3, 1). Find the co-ordinates of B. Sol. Let the co-ordinates of B be (c, B). c Slope of CD =2 Slope of AB which isLtwCD=~3 Equation oF AB the Tine through A with slope ~3 is y-1 043) | Point-slope form or 3x+2y+7=0 A) Equation of CD is 2x ~3y-4 0 alli) To find M. js i, —_e ot og pect yee Solving (i) and (i), yes gta rerg oF een by=-2 Co-ordinates of M are (— 1, ~2) But M being the middle point of AB, has the co-ordinates ( a-3 Bea 2 1 and 2 2 ‘THE STRAIGHT LINE 87 #1 and Hence the co-ordinates of B are (1, - Example 26. Obtain the co-ordinates of the fect of the perpendiculars drawn from the origin upon the lines 3x-Sy+2=0 and 4x-3y+5=0. Also show that the equation of the line joining these feet is 26 + S3y = 11. Sal. 3r-Sy42=0 old) Equation of any line 4 to (()is Sx +3y+.24=0 Ifit passes through the origin (0,0), 4 =0 Equation of the line through the origin 1. to (i) is Sx +3 alli) Solving (é) and (éi) by cross-multiplication. x 1 5 0-6 10-0 9425 7" “7 * Footot fom On cDieM (~F a 7) Now Ar-3y+5=0 soli) Equation of any line .L to (iii) is 3x +4y += 0 It is passes through the origin (0, 0), 4 = 0 <+ Equation of the line through the origin 1 to (iii) is 3x + 4y =0 wid) Solving (if) and (jv) by cross-multiplication, or x= —_._y__t_ 0-20" 15-0 iss ++ Footof.t from 0 on (iti) is N -4, 3 Equation of the line joining M and Nis y Two point form or y 5 . 78 or oe (#7) ie. s3y-% 8S ae a or ree + 53y =e or 26x+53y=11, Example 27. Find the equation of the line joining the feet of the perpendiculars drawn from the origin to the lines x+3y #20=0 and 2x-y-10=0. Sol. Please try yourself. [Ans. 2c —3y—14 = 0] Note. Cireumcentre. It is the point of concurrence of the right bisectors of the sides of a triangle Example 28. Find the co-ordinates of the circumcendre of the triangle whose vertices are (~ 2, 2), (2; — 1) and (4, 0). 88 GOLDEN CO-ORDINATE GEOMETRY Sol. Let A(- 2, 2) ; BQ, — 1), C(4, 0} be the vertices of AABC. AR2,2) Let the right biseetors of BC and CA meet in P. Then P is the circumcentre. To find the equation of LP, the right bisector of BC. Co-ords. of L, the mid-point of BC are Mt, 1), Gaueae) 8 Slope of LP which is 1 BC = -2 @-0 Equation orursy-(-5}+-26-3 or Qytl=-4r412 or 4x4 2y—-11=0 ld) To find the equation of MP, the right bisector of CA. Co-ords. of M, the middle point of CA arc, (4? 22 Vie. (1, 1) 2° 2 1 442° 73 Slope of MP which is LCA Equation of MP is ye Slope of CA = (1) of 3x-y-220 en) Circumcentre Psis the point of intersection of LP and MP Solving (#) and (ii) by cross-multiplication, x —_ =334+8 212 Note. For another method, sec Chapter I. Example 29. Find the incentre of a triangle formed by the lines yo 1S e0,3e+4y=0 and I2y-Sx=0, Sol. Let yo15=0 3r4dy=0 12y-Sr=0 bbe the equations of the sides BC, CA and AB respectively. Solving simultaneously, taken two at a time, the co-ords. of the vertices of the triangle are A (0,0) ; B (36, 15) ; C(-20, 15) a= BC = VG6+ 20) + (15 - 15) = V(S8F = 56 beCA =V(-20- 0)? + (15 -0)' =V4004 205 = V6I5 = 25 c= AB a V(36) + (15) = VID00+ 25 = VISE = 39 Hence the circumcentre P is { 3 3) . y-15=0 c ‘THE STRAIGHT LINE 89 + Co-ordinates of the incentre are given by any + brz+e%y _ $6{0) + 25(36) + 39(- 20) _ 0+ 900-780 _ 120 _ | xe atb+c 56 +25 439 ~ 120 120 = Dat bya + Ws 56(0) + 25(15) +39(15) _ 0+ 375 +585 _ 960 ab+e 5042543920 120 Hence the co-ordinates of the inceatre are (1,8). Example 30. Find the incentre of the triangle formed by the linex cosca+y sina.=p and the co-ordinate axes Sol. The equation of the tin: is y xeosa+ysina =p -) ‘The equations of the co-ordinate axes are yal ii) =0 Co-ordinates of O are (0, 0). Solving (ii) and (i), co-ordinates of Aare ( a 0 ) ma Solving (8) and (fi), co-ords. of Bare(o os 7 T Also AB= (ska-e] +(0-38a] cosa Sina Vv. ee Femaror ap cota’ sina costasina Cosa sina 2 ons sina coset Co-ordinates of the incentre are given by wate) +20) wosasina "sina cos —P__,02-,k_ cosasina sina” cosa . O+p +0 . Pp “ptpeosat+psina T+cosa4sina tea torte in | sina 2, ,L cosasina ” Sina * cosa ete Pp. p+pcosa+psina 1+cosce+ sina Hence the co-ordinates of the incentre are | ———2—_ , ____P ___. Trecosa+sina’ 1 +cosatsina ct GOLDEN COORDINATE GEOMETRY Example 31. Find the area of a triangle formed by the lines x4 4y = 9, 9% 4 10y +23 = O and 7x + 2y =H. Sol, Let x+dy-920 nfl) 9x + Wy +23 =0 Fes 2y—11=0 bbe the equations of the sides BC, CA and AB of AABC. From (ii) and (ii), —~—~» —2—. Co-ordinates of A are (3, = 5). 2 - x ¥ 1 From (if)and (il), aig" eet Dew a ae on eliys or yet Dept ig 1 tye? 2 Co-ordinates of B are (1,2). 3 5 m x a ~ From (and (i, 55° 45° Teas on, xy 2 * 782 * [104 ; ao + Co-ordinates of Care (- 7,4) >< Area of AABC = 1 (645444 14435 —12)= $652) = 26. Example 32. Show that the area of the triangle formed by the lines whose equations are y = m,x 4c, gl Cire? ya my + e;and x= is. Sol. Let yemxrte; Yr mx +e, x20 - be the equations of sides BC, CA and AB respectively of AABC. Solving (ii) and (if), x=Oy=e) Co-ordinates of A are (0, c3). Solving (iii) and (A), x=0,y=e Co-ordinates of B are (0, ¢1) To solve (i) and (ii) ; subtract (i) from () O= (om + mx + ¢)—c2 or (m-m)x= = MICA MCD + ney — MHC _ Mey - MCD m= my my= may Putting this value of x in (i), ey ey macy = me. Co-ordinates of C are | <2. me m= my" m= my THE STRAIGHT LINE a A _ exler- en) | cals Eher= Od) Cher Ca) my =m; Area of AABC L cre L =F. (e2-¢; B mm 2-8) 3 [in magnitude] Article 2.23. Condition of concurrency of three lines. Sol. Let the lines be aux + by +c; =0 at + by +e, =0 ay tby te= The three tines are concurrent if the point of intersection of any two of them lies on the thind. cae 6 ‘ yo. Solving (i) and (i), we set = = oa ayen = 303 as= a; or re Point of intersection of (ii) and (iti) is (See apy=ayhy" ay =a] If this point Jies on the line (j), then bycy- ber aya _ “(oe +O Tym ayb, | T=? or 4 (bye) — Brea) + By (avez — a3¢5) + €\(agbs —aybg) = 0 which is the required condition, Example 1. Find the value of k so that the lines x —2y + 1 = 0, 2x—Sy + 3= 0, Sx ~4y +k = Oare concurrent. Sol. The equations of the lines are x ~2y + 1=0 2e-Sy 4320 Seay +k=0 Solving (i) and (i), we have | — y ogi or or xeLyel =7 Point of intersection of (i) and (iii) is (1, 1) It the Hines are concurrent, then the point of intersection of (i) and (i) lies on (iif) S()-4(1) +420 of k=-1. Example 2. Find k such that the lines 3x ~ 4y ~22 = 0,.x = y +6 and kx + Sy #8 = O.are concurrent, Sol. Please try yourself. [Ans. 6] 2 GOLDEN CO-ORDINATE GEOMETRY Example 3. Show that the straight linesx-y- 1 = 0, 4x + 3y = 25, and 2x = 3y # 1 = Oare concurrent. Sol. The equations of the lines are x~y 4x + 3y-2! Ww=3y+ 150 . 1 Solving (?) and (i), Bes Tass Te or Seth y oF redandy=3 2 The point of intersection of (f) and (i) is (4, 3) Ities on (ii if 2(4)-3G)+1=0 or if8-941=Oorif0=0 whichis tue. the point of intersection of (i) and (ii) lics on (iii), therefore, the three lincs are concurrent. Example 4, Prove thar the lines x =y = 6, dy =x + 22 = 0, 6x + Sy +8 = 0 meet ina point. Also find the co-ordinates of that point, Sol. Please try yourself. [Ans 2,-4)] Example 5. Find the condition that the lines y = mx +c) ¢ ¥ = mzt + Cy; y= mye + cy may be concurrent. Sol. The equations of the lines are m,x—y +e, =0 myx-y te, =0 myx-y+ey=0 i) i) ji x y 1 Solvi ad (i), —2_.__1_ a8 ( and Teg hey me, — meg” — M+ My the point of intersection of (2) and (ii) is ( a mane) — my My Mm, ‘The three lines will be concurrent if the point of intersection of (i) and (ii) lies on (iif) or if ms(cy — €2) — (mae — mer) + €3(m2— mi) = 0 or if my{eq—€5) + mx{e3 ~c1) + mle, ~ 2) = 0 which is the required condition. Example 6. Find the condition that the lines px + q,y = I, p3x + gay = Land px + qyy = I may be concurrent. Deduce that the three points whose co-ordinates are (py, 4;), (Pz 42) and Py qx) are collinear. Sol. The threc given lines will be concurrent if (Pid2 = P21) + (P23 = Ps) + (Pa Pits) =O ~O | Prove it as in Example $ se PY az ‘The points (pr, 41) (Pa gz) and (ps, q:) will be collincarif the area of the — ot Ss Pr ct triangle formed by them is zero. ie, if $1@r42—paai) + r4o~ Po) + Pats - rts) = 0 i it 310] =0 fo of Ol hich is tue. Hence the result. ‘Example 7. Show that the straight lines (b + c)x + ay = d, (c+ a)x + by = dand (a +B) +ey=d are concurrent. ‘THE STRAIGHT LINE 93 Sol. The three given lines are (b + e)x + ay=d a) (cto) +byad (2) (a+ bye tcyad oof) To find the point of intersection of (1) and (2) Subtracting (2) from (1), (b—a}x + (a —B)y = 0 or (b-ax-(b-ay=0 or x-y=0 o xey From (1), (b+cyrtarad or (a+b+cjred Fed Tebec d atb+e’atbee Ie-will ie on (pit fasbMd ed ig atbte atbte or if tb ey gy ig SbF CL, hich iste. atbtc atbhtc atbhtc Hence the three given Lines are concurrent. Another Condition for Concurrency of three Lines => Point of intersection of (1) and (2) i( Article 2.24. To prove that the lines a,x + by + ¢, = 0,2,x+b,y +e, =0,and asx + by +e) 20 are concurrent if we can find three non-zero constants 1, m, nso that Hayx + byy + ey) + m(ayx + bry +6) + mlx + byy +65) = 9, Sol. Let the lines. axtby+es0 and xt by ters 0 ; ayh+ dk +e, 20 intersect at (4, &), then agh-+ Bok +c, =0 oni) Now consider, (a,x + byt 4 ¢,) + m(azx + bay +c) + nae + by 4x3) #0 Since (i) isan Meat, it must be satisfied forall values of x and y, In paniculr, it must be satisfied forx = handy = Hayle + byk + 1) + m(azh+ bok + 2) + n(agh + bsk +213) = 0 or 1(0) + m0) 4 nash + bik +c) =0 of ()] or nfagh + byk+ c3) =0 or ah + bsk +e) =0 (nao) which shows that the point (fk) lies on the third line ayx + byy +c; = 0 Hence the three lines are concurrent. Note. The test is useful only when |, m, mcan be found by inspection. It is especially so when / © m=.n = 1 which implies that the left-hand members of the three equations add up identically to zero.) Remember. [f the equations of three lines are yy = 0, 1) = 0, us = 0, they will concurrent if lay + mulg + nus = 0 where § m, n are thiee non-zero constants, For the purpose of our problems f= m=. = 1. ‘The condition of concurrency becomes 4, +u, +4; "0° ie. the sum of left hand sides = 0. 94 GOLDEN CO-ORDINATE GEOMETRY Example 1.Show shat the lines (m=n)x + (n= Dy + (1-m) = 0 (n— De + (l—myy + (m—n) =0 i) (= mbx + (m= nly + (nD = 0 i) are concurrent, Sol. Adding the equation (i, (i), and (iii) we sec that the sum of their LH. sides vanishes identically. Hence, the lines represented by (i), (ti) and (iii) are concurrent. ) Example 2. Prove that the altitudes of a triangles are concurrent. Sal. Let the vertices of the triangle be A(t), 1), B(x, yz) and Abaya) Clits, Y3) woods Stope of BC = = . slope of altitude AL [which is 1 BC] [-ve reciprocal] 5 t € Point-stope form 2:72) (5, ¥3) ‘ yayim(e xy) or YO'2= Pa) = ¥iP2 + ¥3) = = 10 = 3) +449) or 482-43) + 102-3) —12 —3) — Pie —¥4) = 0 (i) Similarly the equations of the altitudes BM and CN are (ts — 44) + 4's =P) — 203 - 13) - Ys — 4) = 0 x(a 2) +904 92) 4961 42) Ys —P2) = Adding the equations (i), (if) and (ji), we see that the sum of their L.H. sides vanishes idemtically. Hence the altitudes of a triangle are concurrent. Example 3. Prove that the medians of a triangle are concurrent. (K.U. 19768) Sol. Let the vertices of the triangle be A(x, ¥:)- Bes, ¥2). Crs, y3)- Let D, E, F be the mid-points of BC, CA and AB Ale) respectively, so that AD, BE, CF are the three medians. Apts 2p) Co-ordinates of D are ( 2° 2 Equation of the median AD is F, — ‘Twa point form ya- * (w=) 8 D c 0.499 OD or =x) or CR + 3— 2c) — yy +) + Ze = V2 + YN 29)—K12+ 9) + ed or XC + Ya = 2¥2) —¥ Oe + t= Bey) = Cavs = a4) = (ays =) = 0 -O Similarly the equations of the medians BE snd CF are (3 + ¥1 — 2y2) Hs +t = 2) — Gays — ya) = (avi — 2) = 0 on) ‘THE STRAIGHT LINE 95 and 201 + Ya 2y3) - G4 + 22-283) — Ry —-)— Gar aay) = 0 Adding the equations (i), (i) and (fi), we see that the sum of their L.H. sides vanishes identically. Hence the medians of a triangle are concurrent. Example 4. Prove that the right bisectors of the sides of a triangle are concurrent, (Phi. U. 1977) Sol, Let the vertices of the triangle be A(xy, y:), Bs, y2) Cles, ys). Let D be the mid-point of BC. Sats Wty 2° 2 Co-ordinates of D ( Abe) X03 Slope of the right bisectorof BC [which ist BC] yan E E Equation of the right bisector of BC is r+ ¥s natn . Bs x- 2S] [Point-slope form YO ads ( 2 ) | c - fad ey ¥3) nox aa [2G +) ° or 2y- 2499) =— de or 292-8) - Os —ys') =— 2a = 1s) tag — 1? or 2x(rq~33) + 2902-9) - G2 -13') - OF -yF)=0 a) Similarly the equations of the right bisectors of CA and AB are 2x(t5—21) + 293-4) - G8? —x7) - 37-91} = 0 sali) 2a(1 = 42) + 2901 =) =? =27) - YP = 92) iil) Adding the equations (i), (ii) and (iii) ; the sum of their L-H. Hence the right bisectors of the sides of a triangle are concurrent. Article 2.25, To find the length of the perpendicular from the point (x), y,) on the line xoosa+ysina=p. ies vanishes identically. Sol. Let AB be the given line x cos. + ysin a= so that OM, the perpendicular from the origin on the line = p and ZAOM =a. Let (x1, y1) be the co-ordinates of the given point P. ‘Draw PQ.4 ABand let PQ= p’. Through P, draw the fine A’PB' parallel to AB and produce OM to meet it at N. ‘Then MN = FQ = p' ON=OM+MN=p+p! Equation of the line A’PB' is xcosa+ysina=p +p! As Pose) His omit 2 Cos c+ yy sin a= p +p" Hence p” =; cas a+ yy sin a ~p is the required perpendicular distance: 96 GOLDEN CO-ORDINATE GEOMETRY Article 2.26. To find the length of the perpendicular from the point (xy, y;) om the line ax+by#e=0. Sol. Let ax + by +c = O be the equation of the line AB. Yv It meets x-axis where y = 0 ¢ axtc=0 or x=-< @ Co-ordinates of A are ( It meets y-axis where x = bytes0 or ya-£ Co-ordinates of Bare ( - 5 Let P be the point (&, ¥1) m1 ve ii) APAB= 55) Va? +B PL iii) . . ‘waa ¢ From (ii) and (iif), Tab a+b? PL= 555 (an toys te) ant by te PL= . Va'+ ‘Remember. Length of perpendicular from the point (x), y,) on the line ax + by + ¢ = 0 is lan +byi +e] Va + Rule 1. Replace x by x, aud y by yy in the feft hand expression af equation of line (R.HLS. being ze10). 2. Divide it by V(co-ell. of x + (corel. of yy. Example 1. Fine she length of perpendicular from (~ 2, 3) toshe line y = 2x co-ordinates of the foot af the perpendicular. Sol. Let dr-y-, O and also find the “0 @ ‘THE STRAIGHT LINE 97 Let P(—2, 3) be the given point. Draw PLL AB. pus e223=31_1-4-3-3]_ 10 age Vopseip | Vath OS™ P(-2,3) Equation of any line L ABisx +2y +k =0 If it passes through P(—2, 3), then -24+64k=0 of k=-4 Equation of PL isx+2y-4=0 ii) The foot of the perpendicular Lis the pointof intersection of AB and PL, A t B Solving () and (i) x =2,y-= 1. y~2x+3=0 Hence the ca-ords. of the foot of perpendicular are (2, 1). Example 2. Find the length of the perpendicular from the point (2, 3) on the line x + y+ 1 = and also find the co-ordinates of the foot of the perpendicular. Sol. Please try yourself. [Ans. 3VZ, (- 1,0)] Example 3, Find the length of the perpendicular drawn fram the point (1, 2) on the line 3y = 4x ~5. Also find the co-ordinates of the foot of the perpendicular. 7 (33 Sol. Please try yourself. | as-5-( 3) Example 4. fp and p’ be the perpendiculars from the origin upon the straight lines xsec 0 ~y cosec 0 =a andx cos 0 + ysin® = acos 20, prove that 4p? +p? = a? Sal. One line is asec Oy cosec 0 -a = 0 @ (p= length of perpendicular from origin (0, 0) on (i) 12y [Se.- f2y)- 4] _ [5x -3-4y,—4| _ | Sey - 316 - Tx) - 4] V+ 13 ~ 13 = Hens 52] = [20 _ Ls from P on the two lines are equal. Example 17. i/p is the length of the perpendicular from the origin on the line whose intercepts on the axes are and, show that 2 = +h: Sol. Equation of the line whose intercepts on the axes are a and bis Sage 0) p= length of 4 from origin (0,0) on (i) -(0eO=t ye pe 7 Squaring ‘Taking reciprocals z i+ Example 18. Find the equations of two straight lines parallel to 3x + 4y # 1 = 0 and at a distance of 2 units from the point (= 1, 2). Sol. Equation of any line parallel to 3x + 4y+1=0 is 3x+4y+k=0 “0 If @) is ata distance of 2 units from the point (—1, 2), then SED ADE a op fhuae or k4+5=210 +1 k=5 or -15 Hence the equations ofthe two lines are Sx+4y+520 and 3redy-15=0. Example 19. Find the equations of the straight tines passing through the intersection of x~y + = 0 and 2x ~3y +5 = Oand at « distance of? from the point (3, 2). ‘THE STRAIGHT LINE 101 Sol. Any line through the intersection of given lines is @-y+ 1 +AQe-3y +5) =0 or (44-20)r—(1 + 3Ay + (1 +5) 20 wal) If ts distance from the point (3,2) is =, then (1+2093)- (1+ 3)Q)4 (14517 Sh42 Vasarely 5 V1302 4 10242, or 25(5A + 2)? = 49(1322 + 10% + 2) or 1D2-10K=220 or OF—Sk-1=0 or, he 1,-4 Putting these values of } in (i), the reqd. equations arc. 3x—4y+6=0 and dr—3y41=0. Example 20, Find he equations of the straight lines at a distance of 5 units from the origin and passing through the intersection of x -2y-5=0 and 7x +y-S0= 0. Sol. Please try yourself. (Ans, dr —3y-25=0, 3x4 4y—25= 0] Example 21. Find the equations of the two lines which are parallel to the line through (1, 2) and (5, 5) and ata distance of 3 units from it. Sol. The equation of the line joining (1,2) and (5, 5)is yo2eZEZ Qt) or dy 8e3e—3 or 3r-ay45e0 (1) Equation of any line parallel to (1) is Sx —4y += 0 (2) If the distance between (1) and (2) is 3 units, then Heatles or k-S=218 a k=20 or =10 Putting these values of & in (2), the reqd. lines are 3x—dy +2020 and 3x-4y-10=0, Example 22. Find the equations of the lines drawn through the point (0, 1) onwhich the perpendiculars dropped from the point (2, 2) are each of unit length. Sol, Equation of any line through (0, 1) is y-L=m(x-0) or me-y+1=0 wo(1) Ifthe perpendicular dropped from the point (2, 2) 0n (1) is of unit length, then Lema 2 + op matte +1 nt 1 or 3m?@—dm=0 2 m=0,48 Putting these values of min (1), the reqd. lines are y-1=0 and dx—3y+3=0. Article 2.27. To prove that the two point P(x,, y;) and Q(xz, y2) lie on the same side or opposite sides of the line ax + by + ¢ = 0 according as ax; + by, + ¢ and ax + by; + care of the same sign or of opposite signs. 102 GOLDEN CO-ORDINATE GEOMETRY Pp 6 Oa) OY) Q (99) 6.99) ® Fig.(a) Fig.) Let PQ, produced in necessary, meet the given line AB in R. Let R divided PQ in the ratio k: 1 kant Xi at) kel) ket Co-ordinates of R are ( Ries on the given line «(SE ]0( B22) .cun etT kel or afk, +24) + b(ky: + 91) + (K+ 1) = 0 _ __ a tiyte or Kaxy + by; +6) + (ary + by, + e)= 0 or bea byate Case L Ifar, + by, + ¢ and ar, + by, + ¢ are of the same sign, then & is ~ ve and therefore R divides PQ externally ie. R lies on the line PQ produced. the points P and Q lie on the same side of the line as in Fig. (a). Case IL If ax, + by; +c and ary + by; + € are of opposite signs, then k is + ve and therefore R lies within the segment PQ i.e. P and Q are on opposite sides of the given line as in Fig. (6). Article 2.28. Find the equations of the bisectors of the angles between the straight lines ayx+by+ep=0 and ax +hy +e, =0. ‘Sol. The equations of the lines are axtby+e,=0 0) and ax +by+c)=0 valli) The bisectors of the angles between (i) and (ii) are the locus of a point which moves so that its distance from () and (ii) are equal. Let P(x’, y") be any point on either of the two bisectors. ‘Then P is equidistant from (i) and (ii) lau’ tby'+e)]_ lax’ + by’ +o] Vorb? Vay +62 Changing x’ to.x and y' to y, the locus of P is lax+by+e:| — lax+byter| which are the reqd. equations of the bisectors of the angles ‘between (i) and (ii). => ‘THE STRAIGHT LINE 103 2.29, Distinction between the bisectors with respect to the origin : Let the equations of the lines (AB and CD) and be so written that the constant terms c; and c2 are positive, Let PG, ») be any point on the bisector KL of the angle AKD which contains the origin O(0, 0) and PG, y) are on the same side of AB. aye + by +c, and e, are of the same sign. Also O(0, 0) and P(x, y) are on the same side of CD. ax + by + cy and c2 are of the same sign. But c, and ¢2 are both +ve ayrt byte, and ak + by + czare both + ve ay + by ter _ ant + bay +2 Var+b? Vaz? + by is the equation of the bisector of the angle containing the origin. ‘The equation of the bisector of the angle in which the origin docs not lie is at by toy axe + by + cy Vay? +b? Vas + by Example 1. Find the equations of the bisectors of the angles between the lines ()3x-4y41=0 and Sv4iy+4=0 (iy =2x-4 and 2y=x+10 (iii) 2r + 3y+5=0 and 3x+4y+6=0. Sol. (i) The equations of the bisectors of the angles between the given lines are Qeady+_ See layed 4. Sendped Seo iy +4 the equation Vrie ~* veri Ss 3 or 39x—S2y + 13 = (25x + 60+ 20) ie. l4x-12y-7=0 [taking + ve sign] and Gar + 8y+33=0 [taking —ve sign] ie, rs sseo} and 64r+ 8y +3350 Gi) Equations of the bisectors of angles between the given lines are eny-4 x= 2yel 4, De-y-4_ r= 2v+10 v4aeT vied vs vs (e-2y+10) be, xt y-1420 [taking + ve sign] [taking —ve sign] or and Gif) Equations of the bisectors of the angles between the given Jines are 2e+3y+5_ See dvr Vvaeo vO+16 or SQx + 3y + 5)=2VI5 (ar + 4y +6) (VIS — 10)x + (AVS - 15)y + (6 VIS - 25) =0 and QVTS + 10)x + (4VTS + 15)y + (6-VTS +25) = 0. 104 GOLDEN CO-ORDINATE GEOMETRY Example 2. Prove the following facts regarding the lines 15x ~ Hy +1 = 0, 12x + 10y—3 = O and 6x + 66y- 11 = 0 (0 the three lines meet ina point. (i the first two are perpendicular to each other. Gil) the third bisects the angle between the other two. Sol, Hint. (i) The point of intersection of the first two lines is (is 4 ) : 183° 122 ‘Since it lies on the third line, the result follows. (ipSincemmae( 5] —) =~ he msl fotows (iii) Equations of the bisectors of angles between the first two lines are 1Sx-18y+1 | 12x+10y -3 ASx~18y+1_ | 12+ W0y~3 V2i5 +324 * Viabe 100 or Weis * ia or 1Se— ca liyet el2vsidy-3 4, WSe— M41 _ 12e+ 1Oy-3 voxor vax6r 3v6r or ase 18) +1) <2 (24+ Iy-3) or 30r-36y+2= 2 (36x + Wy -9) ie. Gx + 66y~11=0 and 66x-6y-7=0 Since one of them is the third given line, the result follows. Example 3. Find che equations of the biseetors of the angles between the lines 2x + y-3 = Oand x + 2y= 1 = 0, Show that the bisectors are at right angles to each other. Sol. The equations of the lines are 2e+y—3=0 and 3r+2y-1=0 ‘The equations of the bisectors of the angles between them are aut a3. 23tey =-1 a a Sr+2y-1 Vaet T+4 ae Vis or meen pteny5 ie, (QVIS —3v5)x + (WTS - 2VS)p—3VTS + VS =0 and (VTS + 3V5)c + (WTS + 2V5)y -3V13 - V5 =0 Paton -E=B) (2a) ot Hence the bisectors are at right angles to cach other. Example 4. Find the equations of the straight lines bisecting the angles between the straight lines (ax+4y-12=0 and tx +3y-23 20 Gd 5e+4y-7=0 and Sx-12y+7=0. Sol. Please try yourself. Ams. ())x-y—11=0,x + y—5=0, (px + Sy -9=0, Se —y— Example 5. Find the equations of the bisectors of the angles between the lines de-3y+2=0 and 12x+5y- Also show that they are at right angles to each other. Sol. Please try yourself. Example 6, Find the bisectors of the angles between the lines 3x + 4y = 11 and 12x - Sy = 2.and distinguish between them. ‘THE STRAIGHT LINE 105 Sol. Writing the equations of the lines so that their constant terms are positive, we have -ar-dy+ t=O and -12r4+5y+2=0 ‘The bisector of the angle in which the origin lies is = Bur4y+ dd | = lore Sy +2 Voeto ~* Videos or cee THe or 3r-My+19s0 ‘The bisector of the angle in which the origin does not lie is aBr-4y+ tl = 12e4 Sy +2 V¥O+ 16 Vi4d+ 25 or cusgtu Ste Sy +2 or lix43y~17=0. te Remember. If < 45°, tanO <1 10> 45°, tan > 1, Example 7, Find the equation of the bisector of the acute angle between the lines 3x -4y = Sand 5x + I2y = 26. Sol. The equations of the linesare 3x —4y-5=0 and Sr + 12y-26=0 ‘The equations of the bisectors of the angles between them are 3r-4y-5 Sx + 12y - 26 Ax -4y-5 Sx +123 26 "ae o SS + 144 1B or 3% -— 52y- 65 = = (25x + Gly - 130) or Ie — 112y + 65 = 0 ii) [Taking + ve sign] and 64x + 8y—195=0 iv) [Taking - ve sign] Now, let 0 be the angle between one of the Tines say (i) and one of the bisectors say (ii. Slope of (?) = 3; Slope of (iii) leo un coi tanO<1 = O<45* or 20 < 90° (ie. £ BPD < 90°) ie. angie between the lines AB and CD is acute. Hence (iif) is the bisector of the acute angle between the lines. 12 Remember : Whenever distinction is to be made with respect to the origin, write the equations of the lines so that their constant terms are +e. Example. Find the equations of the bisector of the acute angle between the lines 2r=y=4 = 0 and x-2y 4 10 = 0. Also indicate the angle containing the origin. 106 GOLDEN CO-ORDINATE GEOMETRY Sol. Writing the equations of the lines so that their constant terms are + ve, we have -2e+y+4=0 -@ and x-2y+10=0 wali) Equations of the bisectors of the angles between them are a eee or -2t+y+4=2(r—2y+ 10) Taking + ve sign, -2x+y+4=x-2y+10 or 3r-3y+ x-y#2=0 . This is also the bisector of the angle containing the origin. Taking -vesign, -2e+y+4=-x+2y-10 or x+y=14=0 iv) Let 0 be the angle between one of the lines say (i) and one of the bisectons say (tit) slope of ()=2, slope of (i 2-1 1 T+ 3% tanO<1, “ 0<45° or 20 < 90" i.e. angle between the lines AB and CD is acute. Hence (iii) is the bisector of the acute angle between the lines. tan6 = Example 9, Find the equation 10 the bisector of the acute angle between the lines 3x + dy = Oand Sx- 12y = 0, Sol. Please try yourself. [Ans.x + 8y = 0] Example 10. Find the equation of the bisector of the acute angle berween the lines 4x~3y + 2= Oand 12x + Sy-8 = 0, Sol. Please try yourself. [Ans, &r -y=1 = 0) Example 11. Find the equation of the bisector of the obtuse angle between the lines 3x + 4y = 1] and 12x-Sy= 2. Sol. Please try yourself. [Ans. 3c - 11y + 19 = 0 Example 12. The equations of the sides AB, BC, CA of a triangle ABC are 3x + dy = 6, 12x-Sy=3 and 4x ~ 3y + 12 = 0 respectively. Find the equation of the internal bisector of ZA, Sol. Equations of the sides AB, BC, CA are rt 4y-6=0 12e~Sy-3=0 4x —3y + 12 To find the co-ords. of B : w Solving (i) and (ii) or ‘THE STRAIGHT LINE. 107 Co-ordinates of B are ( To find the co-ords. of C ; a _ x yd Solving @and Gi) = ——5=—=5qag* Tacs ee = ye ° = = 156° = 16 "169 4 . 9 39 w Coonts of Care (TE Equations of the imternal and external bisectors of ZA. (ie. angle between AB and AC) are Sut dy -6 Ar —3y + 12 VO+te ~~ VI6+o or 3x4 dy-6= 2 (de—3y 412) or x-7y418=0 and Ws y4+6=0 The vertices B and C lic on opposite sides of the intemal bisector AL while they lic on the same side - of the external bisector AM. Substituting the co-ordinates of Bin LHS. of (iv) 23-7418 whichis + ve Substituting the co-ordinates of Cin LHS. of (iv) 69 (39 23 1 js Te-"( a} 84 + 18 = 22-55 ~ 68 which is—ve. B and Clic on opposite sides of (iv), Hence (iv) ie. x= Ty + 18 =0 is the required equation of the intemal bisector of 2A. Example 13.A sriangle is formed by the lines y = 10, 3x—4y = 2 and Sx + I2y = 5, Find the equation of the bisector of the internal angle opposite ta the side y = 10. Sol. Please try yourself. [Ans. 64x + 8y -51 =O} Example 14.A triangle is formed by joining the points (9, ~4), (~ 11,3) and (J, 2). Find she equation of the internal bisector of the angle at the vertex (1, 2). Sol. Equation of side CA is. @-1) or 3r4dy-11=0 S12) M Fa ea) of Se 12y4 1920 7 A L 8 Equations of the internal and external bisectors of C (ie. (9.- 4) (-1.-3) angle between CA and CB) are eh. Bee or x#8y-17=0 w(t) and &-y-6=0 ww) 108 GOLDEN CO-ORDINATE GEOMETRY The vertices A and B lic on opposite sides of the intemal bisector CL while they lic on the same side of the external bisector CM. Substituting the co-ordinates of A in LHS. of (1) 9-32-17 which is—ve ‘Substinuting the co-ordinates of B in L.H.S. of (1), — 11-24 = 17 which is — ve. A and B lie on same side of (1). => (1) is the equation of the external bisector CM. => (2) is the equation of the internal bisector CL. MISCELLANEOUS EXAMPLES Example £. Given three lines u, wax + by + ¢,=0 (r= 1,2, 3), Provettartheequation ofthe straight line passing through the intersection of u, = 0 and i, = 0 and = 0is —“4 (0 perpendicular ous = 01s Oo ae uy u, yb, — yb; yb ~ abs” Sol. The three given lines are uy = ayr+ by+e,=0 ol) art by + c= 0 (2) (i parallel to uy = Gis + Day +0520 ~-Q) Any line through the intersection of (1) and (2) is yt han =0 afd) ie. (aye + by +c) + Mage + byy +03) = 0 or (ay + Datghx + (b; + Rbgy + (C1 + Aes) =0 ay + Rap Is sope = — (i) Also slope of @) is -2 (A) will be perpendicular to (3) if ( oe (-# mymy=—1 1 © Or if yay + Ragas = — bybs—Absby of aay + Baby) = ~ (0,45 + ys) _ its + Dabs of yay Baby Putting this value of in (4), the equation of the reqd. line is yay bids 4 ages =" Gaye bby” tae BBs i) Please try yourself. (Use m, = ma) ‘Example 2. Find the equation of the line which passes through the point (2, 9) and through the point ‘of intersection of the lines 3x + 4y~ 7 = O.and 5x- 12y +7 = 0 and show that it bisects the angle besween the given lihes. THE STRAIGHT LINE 109 and Sol. The equatious of the given lines are 3x4 4y-7=0 (a) Se—12y+7=0 (2) Equation of any line through the point of intersection of (1) and {2) is Gx + 4y=7) 45x = 12y +7) =0 (3) If it passes through the point (2, 9), then (6 +36-7)+(10- 108+ 7)=0 or wah Putting this value of 2. in (3), we get &r—y-7=0 =A) Also the equations of the lines bisecting the angles between (1) and (2) are eet. ee or 13@x+ 4y—7)5 2 5(5x— 12947) 348-920 .(S) and. &e-y-7=0 --(6) Since (4) is the same as (6), hence the result Example 3, Find she equations of the lines through the origin which trisect uke portion of the line x $ = 1 imercepted between the axes. A (a,0), E+ (0,6). y, Sol. Let the line + = 1 meet the axcs in A and B, then Let P and Q trisect AB. (0,0 P divides AB in the ratio 1: 2. <+ Co-ordinate of P are 10) +20 10)+200)) ,. (2a b (Oe 88) te (F9) (0, 0) vi 2 (0, 0) Similarly the co-ordinates of Q which divides AB in the ratio 2: 1 are ( Using two point form : b big Equation of OPif = y-0=-2—(r-0) or 2ay=by 4 Bio Equation of OQis y= 2A—Ge-0) oF aye. y-0 3 Example 4. The intercepts made by a line on the axes are in the ratio 2 : 3 and its distance from ihe origin is VIB 5 find its equation. 110 GOLDEN CO-ORDINATE GEOMETRY Sol, Letthe intercepts be 2a and 3a ; then the equation of the line is = + 2-= 1, 2a* 3a or 31+ 2y-6a=0 (1) If the distance of (1) from the origin (0, 0)is VTS, then, 0+0- 6a] _ : Bo, B Seep OMS > Gaee13 or a= Fors Putting these values of a in (1), the reqd. tines are 3e+2y—13=0 and 3x42y+13=0. Example 5. Find the locus of a point whick moves such that its distance from the point (a, 0) is equal 40 its distance from the line x + a= 9, Sol. Let P(r, ») be any point on locus. By the given condition, W(r-a)?+y? =2 44 veo or (ay ty?= (tay or y= 4 al - (xa)? or y? = 4ax which is the reqd. locus. Example 6.A moving straight line passes through a fived point (h, k) and meets the axes in P and Q. Ifthe rectangle OPRQ be completed, find the equation ta the locus of R. Sol. In any position of the moving line through A (ls, A), fet its imercepts on the axes be a and b. ‘Then the equation of the line is e+ fe 1 Since it passes through the point (#, k) in all positions wy key wl) ab Let the co-ordinates of R be (x, »). Then x =a andy =b k y Example 7. The line ax see 0 + by cosee Q = c cuts the axcs in variable points A and B for different values of 0. If OAPB be a rectangle, find the locus of P. ac by cos 0 * Sind From (1), 4 which is the reqd. locus. Sol, Equation of the line is =e ao cox 0 * xin ab b In any position of the moving line sin ecosO OAS s0B= It the co-ordinates of P are (x, y), then os 0 a Dy or O varies, the reqd. locus is obtained by climinating @ between (1) and (2). From (1) and (2), acsecos®, byscsind Squaring and adding, ax? + BV? =e? (cos? 0+ sin? 0) of ae + bP =c? isthe reqd. locus, . ‘THE STRAIGHT LINE i Example 8. Find the locus of the foot of perpendicular from, the origin on the tine which always passes through (h, K). Sol. Equation of any line through (/, &) is y—k=m(-h) (1) 1 (hi) Slope of any line perpendicular to (1) is > “ The line through the origin perpendicular to (1) is y-02-4¢-0) or my+x=0 (2) “im varies the reqd. locus of M, the foot of the perpen- © x dicular, is obtained by eliminating m between (1)and (2). x From (2), man%, @, y Putting this value of min (1), we have y—K pe or yaks he or x+y —he—ky=0 is the reqd. locus. Example 9. Find the image of the point (3, 8) in the line x + 3y=7 = 0. Sol. Let x43y-7=0 0) P43, 8) be the equation of the line AB and P, the point (3, 8). Draw PR 1 AB and produce it to Q such that RQ = PR. Then Q is the image of Pin AB. Let co-ordinates of Q be (ci) Equation of ony line 4 to AB is 3x-y += 0. A R 8 Itit passes through PG, 8), then 9-84+2=0 or he Equation of PR is 3e—y-1=0 wil) Solving () and (i), = Co-ordinates of R are (1,2) Also R is the mid-point of PQ S43. ond Bx8., or ae-t ant p=-4 Co-ordinates of Q are (- 1,- 4). 2.30. Polar Co-ordinates. This is another method of representing a point in.a plane. Let Obe a fixed point and OX, a fixed straight line through O. The positive direction of OX is ih ited by the arrow head. The fixed point O is called the pole or the origin and the fixed straight line OX is called the initial line or the polar axis. The position of a point P will be known if ZXOP and the distance OP are known, The length OP i called the radius vector and isusually denoted by r. 2XOPisealled PUr, a) the vectorial an gle and is, denoted by ©, The numbers, and , taken inthis very order, are called the polarco-ordinates of Pand the point Pis referred to as (r, 0). The radius vector is considered positive if ineasured from the pole along the line bounding the vectorial angle and negative if measured in the opposite direction. 0, the vectorial 0 angle is + ve or—ve according as it is measured inthe anti-clackwisc or clockwise direction. Qa, By 112 GOLDEN CO-ORDINATE GEOMETRY Example. Plot the point ( -4 ¥] . 2 line and ZXOZ, == = 120°. Since radius vector is negative, cut off OP = 4 units of length Sol. Let OX be the ini from ZO produced {starting from O}. Then P isthe point { -4, 2). . P(-4, 120°) Article 2.31. Relation between Cartesian and Polir Co-ordinate of a point. Let us assume thatthe pole O.and the initial line OX in the polar systema are respectively the origin and the x-axis in the cartesian system, Let (x, y) be the Cartesian co-ordinates and (7, 0) be the polar co-ordinates of a point P. Drw PM 1 OX From nt. 4d. AOMP, we have % cos . xercosd AD) 2esino ysrsino -i) Oo x ™M x These equations express the cartesian co-ordinates of a point interns of its polar co-ordinates, ‘Squaring (f) and (ji) and adding, we bave 2x? +? = F(cos’ 0 + sin? 6) =F" veey nit) Also cos == and sind=% Ai) These equations express the polar co-ordinates of a point in terms of ifs Cartesian co-ordinates. ‘Example 1. What are the Cartesian co-ordinates of the points : z ip (2.2 (+3) w (-23). Sol. (i) Let (x, y) be the Cartesian co-ordinates of the point ( 4, 3) Here r=4,0= 5 x L M X=rcosO=d4cosF=4x 522 Ie yarsin 0 = 4 sin Fa 4x the Cartesian co-ordinates are (2, 2V5). (id) Let (% y) be the Cartesian co-ordinates of the point ( ‘THE STRAIGHT LINE 113 m oe x=rcos0 = —2tos = . =-2sinZ=- 2x4 yersinO=—2sin E=- 2x5 c+ the Cartesian co-ordinates are (V3 ,~ 1). Example 2. Convert P (2 5 ) 32 (3 3) imo Cartesian rectangular co-ordinates and hence write the length of PQ. Sol. Let the Cartesian rectangular co-ordinates of P and Q be (ty, ¥) and (ro, 2) respectively. Here 7” =2,0, 10s 6; = 2.cos yy 7, sin 0, = 2 sin x Also n3G=5 Xy=72.C08 03 = 3cos $= 3(0) =0 y2 rysin 0; =3 sin =3(1)=3 P + (v2, V2) and Q + (0, 3) PQ= Vivi - 0) + (VI - 3) = V2+249-6v2 =VI3-6V2. Example 3. Find the polar co-ordinates of the points : OCL-D i) Ch) Sol. (i) Let (7, 0) be the polar co-ordinates of the point (~ 1,~1) Here xe-Ly=-1 raVieey sviFl ovo 1 reos0=x f $0802 —Fe Oe) rsinOzy x sin0=-Je(-vo) lies in third quadrant. x x Sa tan @ tstn=un (x23) 4 5x ony the polar co-ordinates are ( 7 4 GOLDEN CO-ORDINATE GEOMETRY (if) Please try yourself. | Aus. (7 , 3) | (iii) Let (¢, 8) be the polar co-ordinaies of the point Here ree “ reer y eV oe =W=3 cos 052 —F (-we); sin d= 2a (+ ve) Olies in second quadrant. x x 3x : an tn@=—-12—-unteun af) om 08 th poe co-rtanes are (3,2), Example 4. Change to polar co-ordinates the equations @xcosatysina=p (id) xP - ? = 2ay (iit) x? +? = 3ary (i) +P =e ee -y). Sol. () Puttingx =rcosO and y=rsinQinrcosa+ysina=p, we get reosOcosa +rsinOsina=p or cos (0—a) = p which is the required equation. (ia) Putting x = r cos 0, y = sin 0 ins? —y? = 2ay, we get P cos? 0-7 sin? 0 = 2ar sind or P (cos? O—sin® 0) = 2ar sin'® reos 20 = 2a sin @ [= cos? @—sin?@ = cos 20 which is the required equation. (iii) Putting x = r-cos 0,7 = rsin O inx? + y" + 3axy, we get F cos O +r’ sin O=3a.rcos0.rsind or P (cos’ @ + sin* 0) = 3 ar* cos 0 sin O or 1 (cos* 0 + sin® 0)=3 a cos 0 sin® . _3acosOsinO : ‘ * rege aint g Whichis the required equation, (iv) Putting x = r cos 0, y =r sin, 7 +y? =? in? + 7f =a"? -y’), we get (PF = a(- cos? O- 7 sin* 8) or r'=a’r (cos*@—sin? 0) or P =a" cos 20 which is the required equations. Example 5. Change to polar co-ordinates the equations @x' =y¥@a-y, OC +¥-aP eee +y). ‘THE STRAIGHT LINE 15 or or or or or or or or or Sol. (i) Puttingx = r cos 0, y = r sin @ in the equation x° = y"(2a —x), we have cos 0 =F sin 0 (2a—rcos0) cos’ 0 =r sin? 0 (2a—r cos 8) F (cos* 0 + sin? 0 cos 6) = 2a sin? cos 0 (cos? 0 + sin? 0) = 2a sin” 0 cos 0 = 2a:sin® 6 which is the required polar equation. (ii) Putting x =r cos 0, y =7 sin 0, x" +y? = 7°, in the equation (x? + y° —ax)* = a°(r° +)7), we have (P-ar cos 0} = a. 7 Pr-acosdy sa’? or (r—aeos0)'= a? r-acosQ2a or r=a(1+cos0) 1=2acos? 0/2 which is the required polar equation, Example 6. Change to Cartesian Co-ordinates the equations : () rcos(@-a)=p (i) P = a? cos 20 (iii) r = a sin 20 (iv) P sin 20 = 2a? ()r=asecO + btan 0. Sol. () ros (0-a)=p r(cosO cosa+sinOsina)=p or reosGeosa+rsinOsina=p Putting cos @ = x/r, sin 0 = y/r, we get Xcosa+ysina=p whichis the required equation. i P=a?cos20 or =a" (cos*@-sin? 0) Putting cos 0 = ©, sin % we get Pee (E-5 or rs a(?-y’y Putting 7? = x7 + y7, we get (x +7)? = a°(7 —y*) which is the required equation. (i) rsasin20 or rsa.2sinOcos0 Putting cos=£,sin0=2,weget r=2a.%.E ot P= 2ary Squaring r= 4a’?y? Putting 7 =27+)7, weget (x? +y?)? = 4a7x7y? which is the required equation. (iv) P sin 20=20? or 7. 2sinOcosO= 2a? Putting cos © = x/r, sin @ = y/r, we get xy = a" which is the required equation. ” reasecO+btan® ot r=——+b1and 080 Putting cos@=%,und=,wegea reZ+b.% or rx fy r x Er air r qeeratby or r(x—a)= by Squaring, P(x —a) = by? Putting PF =x +7, weget @7+y\e-a)' =O)? which is the required equation. Example 7. Transform the fotlowing equations 6o Cartesian co-ordinates : 2a @r (i)? 2a"? cos 02 (ii)? cor a’? 116 GOLDEN CO-ORDINATE GEOMETRY Sol. Putting cos 0 =~, wegel r—x=2a ot r=x+2a Squaring, 7 = (x+ 2a) Putting P=x7 +57, weget x*4y?=(v42a)? oF y?=dar4da? or y?= dale +a) which is the required equation. (jt = 2 cos Squating, r= a co? or raa(i+P*)ordreasacma Putting cos © = © , we get eae or Wears aor —ax= ar ‘Squaring (2? ax) 2a? Putting, ? =x? +y",we get (x? + 2y? ary? =a? ("+ y%) Which is the required equation. (iit) Please try yourself. [Ans. 7 = da(a —x)] DISTANCE FORMULA Article 2.32, Find the distance between two points whose polar co-ordinates are given. Sol. Let AQ, 8,) ; B(rz, 02) be the given points. Alt, 4) Join OA, OB. Then OA = r;, OB =r / 2XOA = 0), ZXOB= 0, ZBOA = 0, -0; In AOAB, by cosine formula, AB? = OA’ + OB?- 20A . OB cos BOA a4) + ry? ~ 2r,r2 00s (0; — 0) Hence AB = Wr,” + r3° = 2ryrz cos (8; - 03). Example. (i) Find the distance between the points (« 3) (> 5 ) 2 (ii) Find the distance between the wo points whose polar co-ordinates are (a, a) and (~ a, + 4). What do you infer from the result ? (ii) Show that the points (0, 0); ( 4 3) : (+ 3) form an equilateral triangle, (i) Prove that the lines joining (0, 0) : (2 5) aad ( 3 |lerm an equilateral triangle. Sol. (i) Required distance =V a? + (3a)? = 2(a) (3a) G3 -¥) VF 90 GaP c08F = 1002 6a? 4 V0 = 3aF = aT. ‘THE STRAIGHT LINE 117 (ii) Requited distance =Va4(- ay 2a, (-a)eos(a+a-a@) Vets a + 2a conn = Via 26? I Hence (a, ct) and (= 4, x + a) represent the same point. (ii) Let 00, 0) ia(s 3} (+3) be the given points OA =0B=4 a. A (4, 2nd3) (4, 13) Ve ee 24.aeos( ) a ‘4 = 32-3208 % «OAS OBS AB Hence OAB is an equilateral triangle. AB 1 22-32%) eve es 2 32%3 16, 0) R (iv) Please ary yourself. Article 2.33. To find the area of the triangle whose vertices are given in polar co-ordinates and to deduce the condition of collinearity of three points. Sol, Let O be the pole and OX be the initial linc, Let ABC be the triangle and (1, 0)) ; (F3, 02) ; (rs. Os) be the co-ordinates of the vertices A, B, C respectively. Join OA, OB, OC. Then OA =r, OB =r, OC =ry £XOA = 0), 2XOB= 0; £X0C +0, Now SABC =AOAB + AOBC-AOAC _...(i) Since the area of a triangle = 4. Product of two sides x sine of the included angle 2 x AOAB = 1.04. OB. sin AOB = 3 r,r25in (0 ~0)) AOBC = OB. OC sin BOC inn sin (0; -0,) AOAC # } OA. OC sin AOC =! 75 sin Os ~8)) From (i), AABC = 3 [rpry sin (O; —0,) + rors sin (05 — 93}—rirs sin (Os —0;)] = § [rvra sin (@ —0)) + rors sin (03—0,) + rary sin @; ~Os)} [- sin(-0)=-sin 9] ot in the form of a determinant, ryeosO; rysind, 1 BABC=+] r,00802 rsin 1 ryo0s0; rysin@; 1 Note. The co-ordinates of points A, B, C when replaced by cartesian (rectangular) co-ordinates become (ry cos:Op, r1 sin O;) 5 (72.008 9, 79 3iN 03) ;(r3-cO8 Os, r3 in 03). 118 GOLDEN CO-ORDINATE GEOMETRY no0 rycosO, rysinOy 1 My 1 [gives A==|rycos8; rysin0; 1 yd rycos83 r3sin@s 1 The points A(r, 0;) ; Brae 02) ; C(ras Qs) will be collinear fie, will lic on a straight line] ifthe area of the triangle assumed to be formed by joining them is zero. ie, TyFz Sin (82 ~ G)) + rary sin (83 — 63) + ry7; sin (8; ~ 83) = 0. Example 1. Find the area of the triangle whose vertices are 0@% 246+5);(300+7) ¢o(-2 F): (43): (-+3): = 2a, 0: = 0+ E15 = 34,055 0+ rira sin (@; —}) + rors sin (03 — 02) + rary sin (0 —O5)] a+] 20? sin 2 + 6a? sin % i =5[¥ sin 5 + 6 sin +3¢in( - Sol. (i) Here ry = a, 0 Area of triangle Af aq? YE YS 4g n3[4 Pre Fe S| ¥5a? 3)_ Ya S_Ssv5 mal 33] pa (i) Here r; == 2, 6; = * nel, Loe Fin==1, 02% Area of triangle = } [ryrzsin (8; —6,) + ryrs sin (05 — 03) + 797, sin (©, - 05) “[e ania 3- F)+ an nsa(Z F}ee ne aan Bs =y[-290(-F]-sin(-3} +20] 320i Fc s 2sin( x3) =3[250(*-$]+ 8 ~2an$] 3 [aan oF 2sng] e. Example 2. The vectorial angle of a point P om the join of the poims P,(rs, 0;) and P(r, 03) is 0 +02 sh SLE: | Show that the length of the radius vector of Pis 2 cow os * Sol. Let the co-ordinates of P be (r3, 04). @O This requited to find r3. THE STRAIGHT LINE 119 As the points P,, P2, P are collinear, the area of the triangle assumed to be formed by them is zero. 7 $lrarp'sin (82—0)) + r3rx sin (By ~ 03) + rar; sin (8, -8:)] = 0 or rasin@a— op +rzvsin( A550.) ry sin 0,5 Ie of) or rr sin (0) ~ or ra(ry + 73) sin or rs(rat 73) sin we cos moh Article 2.34, To find the equation of « straight line the perpendicular on which from the pole is oflength ‘p’ and makes with the initial line an angle a. Sol. Let AB be the line and OM be the perpendicular from O onit, so that OM = p and ZXOM =a. Let P(r, 6) be any point an the line. Join OP. Then OP =r and ZXOP = 0 Now XOP- £XOM -a From rt. £d AOMP, we have oM =cos(8-a) or or p=rces(0-a) which is the required equation. Cor. 1. Ifthe line AB is perpendicular to the initial tine, shen ast if AB is to the right of O or asx if AB is to the left of O. ‘The equation p= r-cos (0—«) becomes p=rcos 0-0) orp=rcos @—x) =r cas (4-0) ie, p=rcos0 or p Cor. 2. If the line AB is parallel to the initial line, then a=5 if AB is above the initial fine B scos (0-a) ros 0. or as = if AB is below the initial fine ‘The equation p = r cas (0 —«t) becomes m an parcos(0-3) or pareos (0-3) ie. =reos/=—0 ie, = reos( 3% ney P= z ey p= 2 ie, psrsind ie, p==rsin@. 120 GOLDEN CO-ORDINATE GEOMETRY Cor, 3. Equation of any line parallel to p = r cas (0 ~a) is p’ = r cos (0a) [as only p will change, while a will remain unchanged] Cor. 4. The co-ordinates of M, the foot of the perpendicular from O, om AB i.e. on. p=rcos(0—a), are (p, «) OM =p, 2XOM =a. Article 2.35, Polar equation of a line through the pole. Let the line of inclination a pass through the pole. ‘Then vectorial angle of every point on the line will be a. If (r, 0) is any point on the line, then 0 = which is the required equation. Example 1. (i) Find she polar co-ordinates of foot of the perpendicular from the pole on the line joining the poinis (vy, 8,) and (ry 0). (ii) Find the length of the perpendicular drawn from the point (r),0;) on the line r cos (0a) =p. Sol. (dj Let (p, ct) be the polar co-ordinates of the foot of the perpendicular from the poke O on the line joining A(r,, 8;) and B(r2, 4). Join OA, OB and OP. OP = pand ZXOP =a Equation of line AB is reas (0—a) =p ani) Aand B both lic on AB ie. (3) ry cos (0, ~a)=p; 7; cos. (0; 17, c0 (0; ~) = 72 c08 (0-«) or 7608 0, cos «4 +r; sin 0 sina = ry 008 02 60s ct + 73 sin O> sin ce or (7 sin 0, ~ rp sin 0.) sin a = (72 cos O2—r; cos 8,) cos «t each =p 12 005 0; - 7; cos 0, nas 2 77 sin 0} = ry sin Op 8, - ry cos 0, stant | BE eae 7sin 0; ~ ry sin 07 Now AOAB® LAB. p=ip.Vr? #77 2772008 (0; - 0.) Also. - OA. OB. sin AOB = 4 ryr2 sin (0; ~0,) Vig + rg = 2ryra cos (0; — 03) = Lrire sin (0; -03) rary sin (8 = 82) Vr? +r = 2ryry cos (8; ~ 8) Hence the polar co-ordinates of P, the foot of perpendicular trom O on AB are (p, (1) ie. ‘THE STRAIGHT LINE 121 (ii) Let AB be the line whose equation is rcos (0-a) =p wai) Draw OL.1 on AB. Then OL =p and ZXOL= «. Let P(r;, 0,) be the given point. Let PN = d be the length of perpendicular from P on. AB, Through P, draw CPD parallel to AB, Produce OL to mect CD in M. LetOM =p" ‘Also LXOM =a Equation of CD is r cos (0-a) =p! As CD passes through P(r, 03) 7 €08(0;- a) =p' =p+d . d= 1, cos (0,-«)—p which is the required distance, Example 2. Show that the four straight lines r(cos 0 + sin Q) = = 1, r(cos 0 —sin 0) = = I make up a square and calculate the lengih of a side of the square. Sol. Changing to Cartesian system, the equations become and ie, fi) salt) so(iil) eniv) Equations (i) and (ii) represent parallel lines Equations (ii) and (iv) represent paralle] lines Also equations (@) and (iti) represent perpendicular lines Also equations (ji) and (iv) represent perpendicular lines . The figure formed by the lines (i), (i), (it), (iv) isa rectangle. Distance between parallel lines (i) and (ii) diffe if V(corft. of x? + (coetti. of y eit) 2 ys Vi+T "v2 Distance between parallel lines (ifi) and (iv) LO yg vit Hence the figure is a square with each side = V2. Article 2.36. To find the polar equation of the line joining the points (r,, 0,) and (F;, 03). Sal. Let Q be the pole and OX the initial tine, Let A(r;,0;) and B(r», 02) be the two given points. Let P(r, 0) be any point on AB. JoinOA, OB and OP, Then AOAB = AOAP + AOPB 122 GOLDEN CO-ORDINATE GEOMETRY £0A. OB. sin AGB =10A.OP. sin AGP +1 0P. OB sin POB or rirasin (0;~0,) = ryrsin (0 ~0:) + 72 sin (0, ~0) or ryrz sin (02— 04) + 77; sin (0, -8) + rr2sin (0-0;)=0 Bia, ) / Dividing by rr,r2, sin (8,-8,) sin(®,-8) sin (@-8,) rm which is the required equation. Note 1. Every equation of the form cos 0+ b sin 0 Lip represents a straight line, the equation can be writtenas f= a.rcos 8 +b. rsin & is Teac thy I which is the equation of «straight line Note.2. Now any line perpendicular to ax + by = Fis ay-br=f or a.rsinO=b. rcosO=1 x reos O=x, rsinO=y or Ew asino=b ood iy (i) represents any line perpendiculato (1). ANGLE BETWEEN TWO LINES Article 2.37. To find the angle between the lines cos (0 —c,) = py and Fr cos (0c) = pz. Sol, Let AB and CD represent the lines reos (0-4) =p, and ros (0 — cg) = py Draw OL and OM perpendicular on AB and CD. Then ZXOL=a),2XOM=a, 3. 2MOL=c-a; Since the angle between the lines is equal to the angle between the perpendiculars to the lines from the origin, + Required angle = a, ~ a2 Hence required angle = |exy— a2 | Cor. 1. The lines are parallel if Jaya Ja=0 ie, a) 203 Cor. 2. The lines are perpendicular if |, ~«: x For Fta Any line perpendicular to r cos (8 — a rons (Bo a; = ay ‘THE STRAIGHT LINE 123 ie, +0-0)] or reos| (0-a)-3] ie, =rsin(O-a) =p! or revs 5-0-a)| ie, rsin(@—«)=—p' or rsin(@-«) Example 1, Show that the equation of the sraight line perpendicular to the line Pz cos-a)+ecos0 is © =—sin(@-a)-esind and the equation of the line parallel to it and through the pole is Q = tax" | — cosate sina Sol. The equation of any fine perpendicular to- 2 x cos@—a) + ecosd is obtained by changing 0 10 5 + 0 and replacing p by a parameter. 2. Required equation is © =| 340-0 | om(3+0) ie. a nsin (@-a) +esin0 -{l) Equation of the given line is e 2 005 (0-«) + ecos 0 or = cos O cosa +sin O sina +ecos 0 ros or = (cosa + €) cos 0+ sina .sin O fi) re Equation of any line parallel to (i) is obtained by simply replacing the constant term p by a parameter. Equation of any line paralfe! to (i) is Pp (cos «a + 2) cos 0 + sin esin® r or P' =r[(cos «+ €) cos 0+ sin sin 0) end) Asit passes through the pole (0,0) >. p’=0 from (i), required equation is 0 =r (cosa + €) cos 0 + sin a sin 0) or sina sin 0 = - (cos +) cos 6 . af _cosate ie, Gotan SZ). sing z ) 23. Sol. Equations of the wo lines. are reas 0=2 ie. reos (0-0) =3 O Example 2. Find the angle between the lines rcos 0 = 2 and 7 cos ( 8 124 GOLDEN CO-ORDINATE GEOMETRY x and roos| @=-= 6 [Comparing () and (i) with r cos (0 =<) =p, cy = 0, tg = 216] Required angle between (i) and (ii) =| 0, a3] = 3. Example 3. Through a given point O a straight line is drawn to eut two given straight line is Q and R. Find the locus of a point P on this variable tine such that @ a 20 Ze (i) 20P = 02 + OR (iii) OP" = OQ.0R. Sol. Take O as the pole and a line OX through it as the initial line. Let the polar equation of the two given lines be Jeacos0+b sind 0) and 42a’ cox +6’ sin Let P(, 8°) be any point on the line drawn through O, culting the lines (i) and (i) in Q and R respectively. LXOP=0' Equation of variable line OP is 0 = 0 (iii) {[Note. To Find the point of intersection of ()) and (iii) we solve the two simultancusly. The value of so obtained ‘gives the radius vector af the point of intersection Q, i, gives OQ], The line (iti) cuts (i), where 4 sacos O'+bsing! ive, cos 6' + bsin 0 wiv) The Line (iii) cuts (i) where sie =a’ cos0 +5 sin! ies qr a e080 +H sino” Also OP=r’ ( Since a ootor 2 ca cast +b sin! +a’ cos! +b’ sin! = (a+a') cos 0 + (6 +b’) sin” Hence locus of P(r, 0') is 2. {a 4a’) cos 0+ (b +5") sin 0, which isa straight linc. Gi Since 20P =0Q+0R

Você também pode gostar